Exam 7

You might also like

Download as pdf or txt
Download as pdf or txt
You are on page 1of 53

‫ م‬10:39 2023/‫‏‬5/‫‏‬13 ASQ CMQ-OE : Practice Test #07 ‫ م‬10:39 2023/‫‏‬5/‫‏‬13 ASQ CMQ-OE : Practice Test #07

1/1* Team development progresses when team members have the basic
ASQ CMQ-OE : Practice Test #07  skills to:
101/159 ‫إجمالي النقاط‬

Manage a project.
Test yourself

Tolerate difficult team situations.

Apply Plan-Do-Check-Act (PDCA) to problem solving. Address storming by norming.


* ‫عنوان بريد إلكتروني‬
Address storming by norming.
sayed.rehan06@gmail.com

1/1* Which of the following best describes how to identify customer 1/1* Part of the overall strategic plan is cost savings through streamlining

wants/needs and should be included on the left side of the QFD house of processes and continually achieving customer satisfaction. Which of the
quality? following would the Quality Department select as the primary quality
initiative to support this strategy?

Ask the research and design team to brainstorm and list customer needs and
wants Create a training program for new hires to ensure promotion of continuous
improvement
Charter a cross-functional team of stakeholders from across the organization, O
including those who design the product, source parts, monitor processes and Champion the lean and Six Sigma culture as a guiding principle for continuous
manufacture the product, to work together to determine customer needs improvement

Develop a voice of the customer (VOC) system utilizing multiple sources like Develop an audit program to verify compliance with the system
complaints, warranty claims, customer surveys, etc. to identify customer needs
Develop a pertinent continuous improvement program with measurable
Make sure the new design considers all past customer complaints regarding the deliverables
old design to ensure that the new product is an improvement

1/1* What would be the first step in developing a company-wide lean


implementation?

Decide how best to deliver training

Draft training materials for use in a pilot project

Analyze the current state

Ensure upper-level commitment to the implementation

https://docs.google.com/forms/d/e/1FAIpQLSe_EAdn9T3cnNgY1MBr4Yi0RS0utxLuIAYLclmRVChiUWq3TQ/viewscore?viewscore=AE0zAgCMmUqMlS… 1/106 https://docs.google.com/forms/d/e/1FAIpQLSe_EAdn9T3cnNgY1MBr4Yi0RS0utxLuIAYLclmRVChiUWq3TQ/viewscore?viewscore=AE0zAgCMmUqMlS… 2/106


‫ م‬10:39 2023/‫‏‬5/‫‏‬13 ASQ CMQ-OE : Practice Test #07 ‫ م‬10:39 2023/‫‏‬5/‫‏‬13 ASQ CMQ-OE : Practice Test #07

1/1* An operations team, considered the "right hand" of the VP of Operations, 1/1* As a key, global quality leader new to the organization, you have been
is tasked with addressing process issues impacting high-profile tasked with implementing a quality strategy for the business. The
customers. This team has access to, and control of, the information flow organization's core business is a pharmaceutical supplier of powdered
being released to the workforce and top management. Because the team goods to customers across multiple countries in Europe, Asia Pacific, and
members are all from Operations, the Quality Director often wonders if the Americas. Past Quality Management System (QMS) performance has
the team is not biased when releasing operational information. Which of caused a loss of confidence for key customers.
the following should the Quality Director choose as an appropriate
action? Top management wants an updated and relevant QMS to recover
customer confidence and enable on-time delivery of quality goods
meeting relevant country and product safety protocols. You are required
Request copies of all process evaluation reports performed by the team to:

Perform a separate process evaluation whenever there are process issues 1. Develop, deploy, and manage a QMS strategy across the global
organization.
Write a procedure on the proper process evaluation to minimize bias
2. Design and implement an effective measurement system that will
Deploy and monitor process performance metrics based on the balanced reflect the performance of the QMS and its strategy.
scorecard

External Opportunities

External Threats

Internal Strengths

Internal Weaknesses

1/1* Patricia runs a music store, selling sheet music, instruments, and
lessons. Which of the following describes money tied up inside the
organization?

Profit

Operating expense

Throughput

Inventory

https://docs.google.com/forms/d/e/1FAIpQLSe_EAdn9T3cnNgY1MBr4Yi0RS0utxLuIAYLclmRVChiUWq3TQ/viewscore?viewscore=AE0zAgCMmUqMlS… 3/106 https://docs.google.com/forms/d/e/1FAIpQLSe_EAdn9T3cnNgY1MBr4Yi0RS0utxLuIAYLclmRVChiUWq3TQ/viewscore?viewscore=AE0zAgCMmUqMlS… 4/106


‫ م‬10:39 2023/‫‏‬5/‫‏‬13 ASQ CMQ-OE : Practice Test #07 ‫ م‬10:39 2023/‫‏‬5/‫‏‬13 ASQ CMQ-OE : Practice Test #07

0/1* A relatively small analytical laboratory is considering ISO certification 1/1* A titanium screw manufacturer is considering implementing new product
due to frequent customer requests. The SOPs that are currently in place tracking software for the production area with the aim of lowering waste
are not as detailed for the most current procedures and some are missing and cost. Before initiating the change, management should conduct an
key sections. The highly skilled analysts have been performing the tests analysis that would:
for quite some time and don't use the procedure manuals routinely.

How should the quality manager begin to ensure the quality Identify the IT employees who will install the software.
documentation meets ISO requirements?
Calculate the revenue generated by the employees for each titanium screw.

Identify new employees that will be needed to manufacture the titanium screws.
Wait until the current procedures expire and then revise and write new ones
Identify each stakeholder that will be directly affected in using the software.
Obtain permission from the CEO to update policies and procedures

Keep the existing procedures; the analysts have proven that they don't need them

Write policy and procedure manuals that are meaningful and useable

‫اإلجابة الصحيحة‬

Write policy and procedure manuals that are meaningful and useable

‫التعليقات‬

Having both meaningful and usable manuals is the first step to ensure that ISO
requirements are met. The problem with unwritten or poorly written policies and
procedures is that they are not subject to the usual organizational reviews and
accountability processes. In the absence of written policies and procedures, different

.approaches make the organization inconsistent and inefficient

1/1* Which of the following analyses involves the determination of a cause


and effect relationship?

Customer retention over the last year, expressed on a per-month basis

Customer attrition rates for three successive years

Profit impacts of customer retention

Average, mean, and median customer retention for each quarter of the year

https://docs.google.com/forms/d/e/1FAIpQLSe_EAdn9T3cnNgY1MBr4Yi0RS0utxLuIAYLclmRVChiUWq3TQ/viewscore?viewscore=AE0zAgCMmUqMlS… 5/106 https://docs.google.com/forms/d/e/1FAIpQLSe_EAdn9T3cnNgY1MBr4Yi0RS0utxLuIAYLclmRVChiUWq3TQ/viewscore?viewscore=AE0zAgCMmUqMlS… 6/106


‫ م‬10:39 2023/‫‏‬5/‫‏‬13 ASQ CMQ-OE : Practice Test #07 ‫ م‬10:39 2023/‫‏‬5/‫‏‬13 ASQ CMQ-OE : Practice Test #07

1/1* 50-As a key, global quality leader new to the organization, you have been 1/1* When planning the implementation of a change that requires significant
tasked with implementing a quality strategy for the business. The new skills and additional software understanding, which of the following
organization's core business is a pharmaceutical supplier of powdered approaches would be most successful?
goods to customers across multiple countries in Europe, Asia Pacific, and
the Americas. Past Quality Management System (QMS) performance has
Provide the new tools and software and have line management make sure the
caused a loss of confidence for key customers. change gets effectively implemented

Top management wants an updated and relevant QMS to recover Communicate the purpose and need for change in advance, train all those
customer confidence and enable on-time delivery of quality goods involved, and ensure follow-up by line management to support change
meeting relevant country and product safety protocols. You are required
to: Communicate the change in advance and have one of the developers of the change
provide on-the-job training and follow-up coaching
1. Develop, deploy, and manage a QMS strategy across the global
organization. Provide detailed work instructions for all those involved in the change and use
follow-ups by line management to monitor and reinforce change
2. Design and implement an effective measurement system that will
reflect the performance of the QMS and its strategy.

A common metrics dashboard for assessing Delivered PPM is set up. It


reports annual levels and trends for each regional business unit. Annual
targets are not the same across the varying business units; they are set
based on historical trends. Will this dashboard format support the
organization in accomplishing this key objective?

Yes, it respects different objectives between business units based on their histories

Yes, the target is SMART based on regional performance

No, the regions cannot achieve their objectives

No, it measures the KPI, but the targets are not adaptive or realistic

https://docs.google.com/forms/d/e/1FAIpQLSe_EAdn9T3cnNgY1MBr4Yi0RS0utxLuIAYLclmRVChiUWq3TQ/viewscore?viewscore=AE0zAgCMmUqMlS… 7/106 https://docs.google.com/forms/d/e/1FAIpQLSe_EAdn9T3cnNgY1MBr4Yi0RS0utxLuIAYLclmRVChiUWq3TQ/viewscore?viewscore=AE0zAgCMmUqMlS… 8/106


‫ م‬10:39 2023/‫‏‬5/‫‏‬13 ASQ CMQ-OE : Practice Test #07 ‫ م‬10:39 2023/‫‏‬5/‫‏‬13 ) ASQ CMQ-OE : Practice Test #07

0/1* As part of the annual business planning process, the Quality Manager
submitted a revised Quality Improvement (QI) plan-with a budget-to be
launched within the upcoming year. To improve the effectiveness of the
total quality system, one al project that was approved is the purchase and 1/1* Organizational functions that support both internal functions and external
implementation of a Quality Improvement System (QIS) software stakeholders are:
package to replace the current system, which contains numerous
database areas and uncontrolled spreadsheets, reports, and other Ql- Marketing and quality.
related documents. In addition, the current system is hard to search,
making it difficult for new improvement teams or auditors to retrieve Engineering and R&D.
specific information.
Information technology and human resources.
The Quality Manager agreed to be the project manager for the initiative.
Finance and sales.
Her financial acumen and general management skills are essential for the
project to succeed. It was estimated that the project would take 30 weeks
from planning to project close.

ROI estimated was a key reason for the project's approval. Two other key
1/1* Which of the following statements best reflects some of the key
factors were that the new QIS would enable better internal and external
components of a strategic planning process?
auditing and serve as a knowledge management system for new
improvement teams. For project ROI to be realized, what are some of the
data that will need to be tracked, controlled, and more precisely Scenario planning, strategic goals, and action plans
estimated as the project is conducted?
Strategic objectives, Hoshin planning, and input from key stakeholders

Auditor feedback after implementation, team labor hours, project resources Guiding principles/values, strategic objectives, and ISO 9001 (QMS level)
consumed, cost of the QIS, hours to maintain the system after implementation
Mission, vision, and strategic goalsV
Team labor hours, project resources consumed, reduced cost of quality, new
customer acquisitions after implementation

User and auditor satisfaction with the new QIS, total project costs, reduction of
recurring quality issues 1/1* Which of the following is an activity sequencing tool that displays task

Team labor hours, project resources consumed, cost of the QIS, reduced cost of
duration estimates and task pathways from project start to finish?
quality, cost to maintain the system after implementation

Work Breakdown Structure (WBS)


‫اإلجابة الصحيحة‬

Team labor hours, project resources consumed, cost of the QIS, reduced cost of Earned Value Analysis (EVA)
quality, cost to maintain the system after implementation
Gantt chart

Critical Path Method (CPM)


‫التعليقات‬

These are the four styles and their order of use as described by Hersey & Blanchard:
directing (for new hires or inexperienced people); selling (for motivated people who lack
maturity or ability but can do more of the task), participating (for people who are able to
do most of the task and may need occasional help), and delegating (for people who are

.)fully competent, know what to do, and are motivated to do it
https://docs.google.com/forms/d/e/1FAIpQLSe_EAdn9T3cnNgY1MBr4Yi0RS0utxLuIAYLclmRVChiUWq3TQ/viewscore?viewscore=AE0zAgCMmUqMlS… 9/106 https://docs.google.com/forms/d/e/1FAIpQLSe_EAdn9T3cnNgY1MBr4Yi0RS0utxLuIAYLclmRVChiUWq3TQ/viewscore?viewscore=AE0zAgCMmUqMl… 10/106
‫ م‬10:39 2023/‫‏‬5/‫‏‬13 ASQ CMQ-OE : Practice Test #07 ‫ م‬10:39 2023/‫‏‬5/‫‏‬13 ASQ CMQ-OE : Practice Test #07

1/1* A Six Sigma team determined that it requires the removal of 0/1* Green Chem (GC) is a chemical distribution company. After 10
organizational roadblocks to make sure the team receives proper consecutive years of increasing revenue and profits, GC unexpectedly
resources. The best Six Sigma role to act on this would be: lost several of its most valuable accounts, resulting in operating losses.
Follow-up interviews with lost customers revealed GC Account Managers
lacked the necessary experience and knowledge to support them in their
Champion.
industries. Top management realizes that they need to improve core
Team leader. customer retention and loyalty and help Account Managers become
experts in customer industries.
Master Black Belt. The VOC diagram shows corrective action and process improvement as 
one of the key VOC information outputs. Customer surveys can and
Process owner.
should be used to support corrective action and process improvement.
?Which of the following approaches will be most beneficial

When customer rates overall satisfaction as dissatisfied, ask the follow up


question: "Why were you dissatisfied?" Use the information from the survey to help
identify root causes for customer dissatisfaction when all surveys have been
completed. Implement process improvements to prevent common or serious
problems.

When customer rates overall satisfaction as dissatisfied, ask the follow up


question: "Why were you dissatisfied?" This information should be forwarded to the
Account Manager so they can follow-up with the customer as soon as possible.

As surveys are completed, have a quality analyst review question scores and
open-ended data for any concerning situations, especially with the super core
segment. A corrective action request can be initiated at the analyst's discretion
and routed to the person the analyst deems best able to address the customer's
issues.

Monitor overall satisfaction scores plus other outcome metrics. If there are any
dissatisfied or failed expectations, ask the customer "Would you like someone to
follow up and work with you to improve your satisfaction?" Affirmative response
triggers a corrective action incident and all relevant survey information is
transmitted to the Account Manager for next business day follow-up.

‫اإلجابة الصحيحة‬

Monitor overall satisfaction scores plus other outcome metrics. If there are any
dissatisfied or failed expectations, ask the customer "Would you like someone to
follow up and work with you to improve your satisfaction?" Affirmative response
triggers a corrective action incident and all relevant survey information is
transmitted to the Account Manager for next business day follow-up.

‫التعليقات‬

This approach gets all of the right information to the right person at the right time. It
confirms customer interest in a follow-up call. It provides data for multiple potential failure
https://docs.google.com/forms/d/e/1FAIpQLSe_EAdn9T3cnNgY1MBr4Yi0RS0utxLuIAYLclmRVChiUWq3TQ/viewscore?viewscore=AE0zAgCMmUqMl… 11/106 https://docs.google.com/forms/d/e/1FAIpQLSe_EAdn9T3cnNgY1MBr4Yi0RS0utxLuIAYLclmRVChiUWq3TQ/viewscore?viewscore=AE0zAgCMmUqMl… 12/106
‫ م‬10:39 2023/‫‏‬5/‫‏‬13 ASQ CMQ-OE : Practice Test #07 ‫ م‬10:39 2023/‫‏‬5/‫‏‬13 ASQ CMQ-OE : Practice Test #07
points that may point to dissatisfaction or unresolved issues across additional aspects
impacting the customer's experience and it puts this information in Account Manager
hands. In addition, this approach ties the customer survey process to GC's corrective 1/1* Different customers have different needs, and your company has
.action system decided to exploit this to capture greater market share. Which of the
following will best help you target product features and trade-offs for
distinct market niches?

1/1* An appliance manufacturer wants to increase customer loyalty. The


manufacturer should: Customer satisfaction surveys and focus groups

Quality Function Deployment (QFD) and Design for Six Sigma (DFSS)
Launch an aggressive advertising and marketing campaign.
Visits and meetings between customers and engineers
Provide the lowest prices on the market.
Market segmentation and Quality Function Deployment (QFD)
Increase the warranty period offered on the appliances it produces.

Implement actions to create a high level of customer satisfaction.

1/1* Which of the following explains why an organization's departments must


be interdependent?

0/1 * Small shifts in a process are more likely to be detected using:


The decisions of one department mainly affects that department

Large sample sizes and less frequent sampling. Organization must be agile and have departments that are autonomous

Small sample sizes and more frequent sampling. Each department has different pay ranges based on the jobs in those departments
and employees must be able to move between departments
Small sample sizes and less frequent sampling.
An organization is made up of various departments that have common
processes, performance metrics, and goals
Large sample sizes and more frequent sampling.

‫اإلجابة الصحيحة‬

Large sample sizes and more frequent sampling.

‫التعليقات‬

Larger sample sizes can cause narrower control limits, making detection of small shifts
more likely. More frequent sampling will show results faster and catch short duration

.change better

https://docs.google.com/forms/d/e/1FAIpQLSe_EAdn9T3cnNgY1MBr4Yi0RS0utxLuIAYLclmRVChiUWq3TQ/viewscore?viewscore=AE0zAgCMmUqMl… 13/106 https://docs.google.com/forms/d/e/1FAIpQLSe_EAdn9T3cnNgY1MBr4Yi0RS0utxLuIAYLclmRVChiUWq3TQ/viewscore?viewscore=AE0zAgCMmUqMl… 14/106


‫ م‬10:39 2023/‫‏‬5/‫‏‬13 ASQ CMQ-OE : Practice Test #07 ‫ م‬10:39 2023/‫‏‬5/‫‏‬13 ASQ CMQ-OE : Practice Test #07

0/1* Company X recently acquired FDA approval for the manufacture of an 1/1* You have been managing suppliers in your company for years, but
innovative medical device product. The CEO decided to shift to a recently the company has made a change in the types of products that
product-based structure instead of the company's current functional they produce to improve profit and competitiveness. Your current
structure. This decision was most likely based on which of the following: suppliers do not carry the components that you need and, with the help of
Engineering and Purchasing, you are responsible for selecting several
new suppliers. Engineering has a list of suppliers that they have been
To lessen redundancy in organizational hierarchy.
using to create prototypes of new products. The engineers purchased the
To maintain organizational cohesiveness between functions. components on their own and did not involve anyone from Purchasing
because the engineers feel that the purchasing process is too
To focus on product production and market factors. cumbersome and slow. The timeline to produce this new product before
competitors start production of a similar product is very short.
To increase internal communication paths.
Many of the suppliers used by Engineering are brokers who purchase the
‫اإلجابة الصحيحة‬
components from suppliers and then resell. Which adverse risk is
To focus on product production and market factors. associated with components from brokers?

Higher cost
‫التعليقات‬
Component availability
Implementation of product production and market factors means providing concentrated

.expertise or knowledge to support the new product's success
Packaging or shipping damage

Counterfeit parts

1/1* A customer whose sales have increased dramatically in the last few
years is requesting more specialized services, including vendor-managed
inventory and just-in-time delivery. You've been asked to come up with a
strategy to keep costs down while providing these value-added services
to the customer. Which of the following strategies would be most useful?

Conflict resolution and activity-based costing

Customer Relationship Management (CRM)

Lean/Six Sigma projects

Quality Function Deployment (QFD)

https://docs.google.com/forms/d/e/1FAIpQLSe_EAdn9T3cnNgY1MBr4Yi0RS0utxLuIAYLclmRVChiUWq3TQ/viewscore?viewscore=AE0zAgCMmUqMl… 15/106 https://docs.google.com/forms/d/e/1FAIpQLSe_EAdn9T3cnNgY1MBr4Yi0RS0utxLuIAYLclmRVChiUWq3TQ/viewscore?viewscore=AE0zAgCMmUqMl… 16/106


‫ م‬10:39 2023/‫‏‬5/‫‏‬13 ASQ CMQ-OE : Practice Test #07 ‫ م‬10:39 2023/‫‏‬5/‫‏‬13 ASQ CMQ-OE : Practice Test #07

0/1* As a result of the latest management review meeting, several standard 0/1* There is a room labeled "Competitive Assessment on the right side of
operating procedures were updated, and employees were trained the house of quality. How should this room be used in QFD?
accordingly. The supervisor was frequently present on the production
floor during the first week following the training to answer questions and
Even if there are no known competitors, this room should be used to provide insight
provide hands-on additional job training to the operators.
on the likelihood and implication of future competition
Which of the following situational leadership styles did the supervisor
To identify major competitors (if any)
model in this situation?
To summarize the relative strengths and weaknesses of products/services
perceived by customers and compare them to competitors

Low task, high relationship To summarize self-assessed, relative strengths and weaknesses of products/
services and compare them to competitors
High task, high relationship
‫اإلجابة الصحيحة‬
High task, low relationship
To summarize the relative strengths and weaknesses of products/services
perceived by customers and compare them to competitors
Low task, low relationship

‫اإلجابة الصحيحة‬
‫التعليقات‬
High task, high relationship
The Competitive Assessment room in QFD should include customer perception ratings for
the company's product/ service features as compared to ratings of competing products/

.services
‫التعليقات‬

In this situational leadership style, the supervisor remains engaged in the task by
explaining and further demonstrating the new way, at which point employees have an
opportunity to ask questions to ensure that they understand the task. The supervisor

.monitors closely

1/1 * Strategic plans may be influenced by legal and regulatory factors like:

Local traffic laws.

The balanced scorecard.

The New York Stock Exchange

The Food and Drug Administration.

https://docs.google.com/forms/d/e/1FAIpQLSe_EAdn9T3cnNgY1MBr4Yi0RS0utxLuIAYLclmRVChiUWq3TQ/viewscore?viewscore=AE0zAgCMmUqMl… 17/106 https://docs.google.com/forms/d/e/1FAIpQLSe_EAdn9T3cnNgY1MBr4Yi0RS0utxLuIAYLclmRVChiUWq3TQ/viewscore?viewscore=AE0zAgCMmUqMl… 18/106


‫ م‬10:39 2023/‫‏‬5/‫‏‬13 ASQ CMQ-OE : Practice Test #07 ‫ م‬10:39 2023/‫‏‬5/‫‏‬13 ASQ CMQ-OE : Practice Test #07

0/1* You have been managing suppliers in your company for years, but 1/1* A key objective of employee management is to continually improve
recently the company has made a change in the types of products that employee competency and employee value to the company. Which action
they produce to improve profit and competitiveness. Your current best aligns with this objective?
suppliers do not carry the components that you need and, with the help of
Engineering and Purchasing, you are responsible for selecting several
Rating an employee's annual performance against five pre-defined levels
new suppliers. Engineering has a list of suppliers that they have been
using to create prototypes of new products. The engineers purchased the
Benchmarking a job's base salary with external companies to set pay rates based
components on their own and did not involve anyone from Purchasing on external factors
because the engineers feel that the purchasing process is too
cumbersome and slow. The timeline to produce this new product before Providing training to help employees perform better in their current position
competitors start production of a similar product is very short.
Providing activities and assignments that build readiness for current and future
The histogram shows data for four suppliers. A scale from 1 to 5 was positions
utilized with a higher score considered best. Based on this information,
which supplier would you select for a component for your new innovative
product?
1/1* Which of the following is the best variable control chart to use if a
Supplier D product takes a long time to produce?

Supplier A
Moving Average Moving Range chart
Supplier B
X and R Chart
Supplier C
X and S Chart
‫اإلجابة الصحيحة‬
Median control chart
Supplier A

‫التعليقات‬

Supplier A is favorable for quality and technical support but has a higher cost. If you add
the scores up, Supplier A is at 15 and if you multiply, they are at 150. Both composite

.scores are higher than the other suppliers, so Supplier A should be selected

https://docs.google.com/forms/d/e/1FAIpQLSe_EAdn9T3cnNgY1MBr4Yi0RS0utxLuIAYLclmRVChiUWq3TQ/viewscore?viewscore=AE0zAgCMmUqMl… 19/106 https://docs.google.com/forms/d/e/1FAIpQLSe_EAdn9T3cnNgY1MBr4Yi0RS0utxLuIAYLclmRVChiUWq3TQ/viewscore?viewscore=AE0zAgCMmUqMl… 20/106


‫ م‬10:39 2023/‫‏‬5/‫‏‬13 ASQ CMQ-OE : Practice Test #07 ‫ م‬10:39 2023/‫‏‬5/‫‏‬13 ASQ CMQ-OE : Practice Test #07

1/1* Survey employees for their input and opinions. Two of the five largest 0/1* You are a quality manager for a supplier of commodity components in
auto manufacturing firms executed a merger that created a new portfolio the automotive sector. Manufacturing is done in Mexico. Your company
of products and expanded existing portfolios of the two firms. One large has incurred large airfreight charges to avoid substantial late-delivery
portfolio serving luxury car customers now contains 12 cars. This portfolio penalties as the result of needing to rework parts which failed to meet
could become one of the most important future drivers of corporate customer specifications. Although there are well-established goals and
growth for the newly-merged firm. targets for product quality, they appear to be ignored. What should you do
next?
Executives need to conduct an in-depth portfolio review to:

Conduct a process capability study to see if the customer specifications are


Eliminate cars that are not at a specified price point. attainable

Add additional cars to expand the portfolio. Analyze results to see if any patterns emerge, e.g., by shift, by day, by customer, by
machine center, etc.
Define and prioritize customer segments.
Establish clear accountabilities for achieving targets
Survey employees for their input and opinions.
Develop a business case for bringing manufacturing back to the United States,
where it is easier to monitor progress

‫اإلجابة الصحيحة‬

Establish clear accountabilities for achieving targets

‫التعليقات‬

There is a possibility that local management is not aware of the significance of corporate
goals and targets. The first step is to make sure that manufacturing management knows

.what is expected

1/1 * What is the process of assimilating a supplier to your business needs?

Specifications

SIPOC

Voice of the customer

Onboarding

https://docs.google.com/forms/d/e/1FAIpQLSe_EAdn9T3cnNgY1MBr4Yi0RS0utxLuIAYLclmRVChiUWq3TQ/viewscore?viewscore=AE0zAgCMmUqMl… 21/106 https://docs.google.com/forms/d/e/1FAIpQLSe_EAdn9T3cnNgY1MBr4Yi0RS0utxLuIAYLclmRVChiUWq3TQ/viewscore?viewscore=AE0zAgCMmUqMl… 22/106


‫ م‬10:39 2023/‫‏‬5/‫‏‬13 ASQ CMQ-OE : Practice Test #07 ‫ م‬10:39 2023/‫‏‬5/‫‏‬13 ASQ CMQ-OE : Practice Test #07

1/1* Company Z has logistics operations in various U.S. locations with 0/1* Company X's newly launched product line and marketing strategy
multiple competitive risks. One of Company Z's strategies, to differentiate resulted in an "overnight success. This sudden growth created an internal
itself from its competitors, is to change its industry image from a low-cost challenge to align the cultural mindset of the workforce from a "mom-and-
provider to a service-driven organization. To achieve this strategy, what pop" garage business into a medium-sized company with a sophisticated
organizational model would best help Company Z achieve its strategy? customer base.

What should the company choose as a starting point to provide guiding


Geographic-based principles for the workforce's alignment with the customers' values and
needs?
Process-based

Functional-based Implement Hoshin planning to solicit feedback from the workforce

Customer/market-based Implement common industry-accepted best practices (e.g., ISO 9001:2008, 5S)

compensate for usual errors in decision making

Delegate the planning to the planners

‫اإلجابة الصحيحة‬

Implement common industry-accepted best practices (e.g., ISO 9001:2008, 5S)

‫التعليقات‬

Implementing industry- accepted standards (e.g., ISO, 5S, etc.) are manageable to deploy
as these standards have been used as guiding principles by many customers, and
therefore will likely align the workforce with the values of the newly acquired customer-

.base

https://docs.google.com/forms/d/e/1FAIpQLSe_EAdn9T3cnNgY1MBr4Yi0RS0utxLuIAYLclmRVChiUWq3TQ/viewscore?viewscore=AE0zAgCMmUqMl… 23/106 https://docs.google.com/forms/d/e/1FAIpQLSe_EAdn9T3cnNgY1MBr4Yi0RS0utxLuIAYLclmRVChiUWq3TQ/viewscore?viewscore=AE0zAgCMmUqMl… 24/106


‫ م‬10:39 2023/‫‏‬5/‫‏‬13 ASQ CMQ-OE : Practice Test #07 ‫ م‬10:39 2023/‫‏‬5/‫‏‬13 ASQ CMQ-OE : Practice Test #07

1/1* By law, XYZ Township has jurisdiction and authority over the operations 1/1* You are the director of quality at a large design/build engineering
and activities of the 36 square miles of property lying within the township company. You have had ISO registration for over 10 years. During your
(this same law defines the jurisdiction of all townships and cities; another most recent audit for registration renewal, you were told by the external
law allows cities to annex portions of townships). When developing its auditors that there were four major deficiencies, any one of which could
strategic plan, which of the following market forces is most likely to be grounds for losing your registration. You were able to take immediate
threaten the success of XYZ Township's plans? corrective action so that your company retained its registration. As a
result of this "near miss," the executive team has directed you to review
the company's quality systems and make recommendations so that this
Occasional tourists from Russia
situation does not happen again. You have 30 days. How should you
ABC Township, which operates on the other side of the state (over 200 miles away) start?

A hunting club that wants to open a 100-acre shooting range in the township
Review whether ISO is still the best quality system for your company, given
The neighboring (and adjacent) city that wants to annex portions of XYZ customer requirements
Township
Focus on increasing the robustness of those systems and processes where
deficiencies were identified

Stress the responsibility and necessity of all department heads complying with ISO
requirements
1/1* A production supervisor often comments that "people are lazy' and is
very suspicious of his staff's willingness to meet daily goals. He tracks Ensure that ISO compliance has executive level support, then cascade ISO
goals and objectives throughout the company
their progress hourly and continually monitors their work breaks. Which of
the following behavior theories is the supervisor using?

Theory Z

Hierarchy of needs

Theory Y

Theory X

https://docs.google.com/forms/d/e/1FAIpQLSe_EAdn9T3cnNgY1MBr4Yi0RS0utxLuIAYLclmRVChiUWq3TQ/viewscore?viewscore=AE0zAgCMmUqMl… 25/106 https://docs.google.com/forms/d/e/1FAIpQLSe_EAdn9T3cnNgY1MBr4Yi0RS0utxLuIAYLclmRVChiUWq3TQ/viewscore?viewscore=AE0zAgCMmUqMl… 26/106


‫ م‬10:39 2023/‫‏‬5/‫‏‬13 ASQ CMQ-OE : Practice Test #07 ‫ م‬10:39 2023/‫‏‬5/‫‏‬13 ASQ CMQ-OE : Practice Test #07

0/1* You are a skilled facilitator who has been asked to help a group of 1/1* In which of the following situations would a team apply Design for Six
managers manage conflict more effectively. What is the preferred method Sigma (DFSS) techniques?
for building the necessary skills?

Specifying tolerances based on engineering standards


Lecture, presentation, and discussion
Determining which new product features are needed
Computer-based instruction
Choosing a manufacturing location
Use a conflict resolution model
Specifying equipment features to monitor and control a process
Experiential training

‫اإلجابة الصحيحة‬

Experiential training 1/1* A large multi-national corporation has a manufacturing division located in
Kansas. It is currently performing well in the areas of profitability,
productivity, and out-going quality. To gage performance of these areas
‫التعليقات‬ with a similar manufacturing division in Canada, they would engage in
Experiential training, perhaps using a scenario- based approach, would resonate with adult what type of benchmarking?
.learners and can be designed to bring out key learning points

Functional

Inter-sector

1/1 * Which of the following is an appraisal cost? Competitive

Internal
Quality improvement

Supplier reviews

Contract/document review

Field performance evaluation

https://docs.google.com/forms/d/e/1FAIpQLSe_EAdn9T3cnNgY1MBr4Yi0RS0utxLuIAYLclmRVChiUWq3TQ/viewscore?viewscore=AE0zAgCMmUqMl… 27/106 https://docs.google.com/forms/d/e/1FAIpQLSe_EAdn9T3cnNgY1MBr4Yi0RS0utxLuIAYLclmRVChiUWq3TQ/viewscore?viewscore=AE0zAgCMmUqMl… 28/106


‫ م‬10:39 2023/‫‏‬5/‫‏‬13 ASQ CMQ-OE : Practice Test #07 ‫ م‬10:39 2023/‫‏‬5/‫‏‬13 ASQ CMQ-OE : Practice Test #07

0/1* You are the Director of Quality and have recently worked with a third- * A company requests that their employees work overtime to finish a large
party auditing group to pursue ISO registration. Due to the prevailing order. The company is offering bonuses in addition to overtime pay if the
culture in your organization and the hard work you invested in designing order is completed on time. How are the employees being motivated?
the quality management system, the organization did well on the audit.
One surprising non-conformance that was issued concerned how
Intrinsically
documents within the quality manual were reviewed and approved. Since
you lead the Quality Department-and because your supervisor (the COO) Esteem building
delegated the quality manual and the quality management system to you-
you signed off on the revisions you introduced. As a result, you now need Increased resources
to revise the process and the related procedure. That said, you are also
Extrinsically
serving as the interim Director of Training, so you need to ensure that the
members of the Quality Department know about and implement the
changes.

When briefing your supervisor (the COO) about the need to train your 1/1* If a customer moves toward a just-in-time (JIT) system of production, yet
team on the proposed revision, the COO asks you if training is the ideal does not inform their suppliers about the change, what is the likely result?
solution to ensure that the members of the Quality Department know
about and implement changes. How would you respond?
The effect depends upon the amount of inventory the suppliers carry

Training is always effective The effect would be limited and only be a problem during periods of economic
downturns
If you employ the principles of adult-learning correctly, training will be effective
No perceivable change; the change to JIT does not affect upstream suppliers
If the training is lively and engaging, the training will be effective
An immediate detrimental impact on the firm's ability to ship finished product
on schedule
Training is not always the answer

‫اإلجابة الصحيحة‬

Training is not always the answer

‫التعليقات‬

Training is not always the answer because root cause behind non- conformance can be
behavioral. In this case the root cause is knowledge-based, so training is the ideal

.solution

https://docs.google.com/forms/d/e/1FAIpQLSe_EAdn9T3cnNgY1MBr4Yi0RS0utxLuIAYLclmRVChiUWq3TQ/viewscore?viewscore=AE0zAgCMmUqMl… 29/106 https://docs.google.com/forms/d/e/1FAIpQLSe_EAdn9T3cnNgY1MBr4Yi0RS0utxLuIAYLclmRVChiUWq3TQ/viewscore?viewscore=AE0zAgCMmUqMl… 30/106


‫ م‬10:39 2023/‫‏‬5/‫‏‬13 ASQ CMQ-OE : Practice Test #07 ‫ م‬10:39 2023/‫‏‬5/‫‏‬13 ASQ CMQ-OE : Practice Test #07

0/1* A final quality inspection plan is in place to verify shipped goods * A project is behind schedule due to a supplier issue and the team needs to
conformance before loading for delivery. The incoming rate of finished take corrective action to meet the deadline. Which of the following methods
goods is exceeding the inspection capability and causing inspectors to would best help them decide where to apply additional funding?
rush. Budget restrictions did now allow for additional inspectors, so a
revised AQL sampling plan is proposed to reduce sampling for some
Work Breakdown Structure (WBS)
inspection checks to save time. Operations management tends to be fast
in putting the blame on inspection if an error is missed and reaches the Gantt chart
customer, so they are skeptical of changing the quality plan.
Program Evaluation Review Technique (PERT)
Which of the following would the Quality Director do to create a more
visible change process associated with the QC plan revision? Critical Path Method (CPM)

‫اإلجابة الصحيحة‬
Tighten the sampling plan on some critical requirements and ease some of
management's concerns Critical Path Method (CPM)

Audit the new quality plan more frequently to assure management that each shift is
conforming to one unified plan ‫التعليقات‬

Use SPC data as an input to the inspection so that inspection could be reduced A CPM chart shows standard cost/duration for each task on the critical path, making it

.easy to see which tasks can save the most time vs. costs to expedite
if process data shows it was running free of special causes

Show quantified error data in Pareto format to show trends before and after quality
plan changes are made

‫اإلجابة الصحيحة‬

Show quantified error data in Pareto format to show trends before and after quality
plan changes are made

‫التعليقات‬

With support from quality engineers, the Quality Director is responsible for establishing
and improving inspection plans to keep them cost effective and responsive to both
customer requirements and process risk control. Management should trust the decisions
made and the Quality Director should check the revised plan using before and after data to
verify that changes made are effective over time. Going forward, this data will also help
.lower management concerns

https://docs.google.com/forms/d/e/1FAIpQLSe_EAdn9T3cnNgY1MBr4Yi0RS0utxLuIAYLclmRVChiUWq3TQ/viewscore?viewscore=AE0zAgCMmUqMl… 31/106 https://docs.google.com/forms/d/e/1FAIpQLSe_EAdn9T3cnNgY1MBr4Yi0RS0utxLuIAYLclmRVChiUWq3TQ/viewscore?viewscore=AE0zAgCMmUqMl… 32/106


‫ م‬10:39 2023/‫‏‬5/‫‏‬13 ASQ CMQ-OE : Practice Test #07 ‫ م‬10:39 2023/‫‏‬5/‫‏‬13 ASQ CMQ-OE : Practice Test #07

0/1* Company ABC restructured its training program by rotating new hires to 1/1* You are an experienced quality manager with lean Six Sigma Black Belt
stay with a department mentor for a week to observe the assembly certification. You were recently hired as the new quality manager at a
process. The Training Steering Committee (TSC) was tasked to evaluate multi-site manufacturing company that is ISO 9001 certified. The biggest
the effectiveness of training given to the rotation teams. Which of the problem is an increasing trend in late deliveries. What action would you
following would the TSC recommend as the effective method of take?
evaluation?

Introduce lean Six Sigma training and form various problem-solving teams
Written test with a required minimum grade
Apply your skills with the Theory of Constraints to identify any bottlenecks that are
Observe and rate how the rotation team cross-trains other teams
causing late deliveries

Presentation of lessons learned to subject experts Conduct a benchmarking study to determine in which quartile your company falls
regarding late delivery performance for the industry and competition
Hands-on performance with expected performance metrics
Audit the product realization process to identify areas and issues relevant to the
‫اإلجابة الصحيحة‬ trend

Hands-on performance with expected performance metrics

‫التعليقات‬ 1/1* Which of the following options describes external factors (and not the
internal factors) that should be considered in a business environmental
Hands-on performance with expected performance metrics is an effective method in analysis?
evaluating the training effectiveness, especially if there are metrics (e.g., rate or error, cycle
time, etc.) The goal is to change behaviors in a way that will drive positive results in the

.short and long term
Product/service realization, technology changes, and customer obligations

Technology changes, competition changes, and community obligations

Customer base, supplier base, and investor obligations


1/1 * Team members are primarily chosen based on:
Technology changes, competition changes, and economic changes

Personal relationships amongst team members.

Nature of the goal.

Seniority.

Training and readiness.

https://docs.google.com/forms/d/e/1FAIpQLSe_EAdn9T3cnNgY1MBr4Yi0RS0utxLuIAYLclmRVChiUWq3TQ/viewscore?viewscore=AE0zAgCMmUqMl… 33/106 https://docs.google.com/forms/d/e/1FAIpQLSe_EAdn9T3cnNgY1MBr4Yi0RS0utxLuIAYLclmRVChiUWq3TQ/viewscore?viewscore=AE0zAgCMmUqMl… 34/106


‫ م‬10:39 2023/‫‏‬5/‫‏‬13 ASQ CMQ-OE : Practice Test #07 ‫ م‬10:39 2023/‫‏‬5/‫‏‬13 ASQ CMQ-OE : Practice Test #07

0/1* One of the first activities of a newly formed kaizen team is the finalization 0/1* You have just started as Quality Director at a manufacturing firm with
of the team charter. As the team charter is being developed, it is multiple locations, with 1500 employees total. You have found that
important for the team to: procedural documentation is lacking and if there is any, it has not been
updated in a long time. There are known issues with too much internal
rework and a lack of planning resources and efforts. Your short list has 25
Use clear wording regarding the purpose, goals, and resources available to the
team. possible areas to fix. The company COO has provided only a general
direction to increase capacity. Not all managers see the same issues, and
Use very precise detail so it can be used to ensure that no changes in direction disagreement is common between departments and up and down the
occur should the team discover new information as it proceeds.
management chain.
Use very general and non-prescriptive wording to allow the team as much latitude
as possible to stay agile. You have a large budget to work with as a large potential customer has
been identified, if you can meet their stringent cost and quality needs.
Request the champion write the charter to ensure that the team meets the The company does not use the usual quality tools, and you were hired to
objectives and goals of management.
help.
‫اإلجابة الصحيحة‬
As your effort progresses, you and other leaders narrow your focus to 12
Use clear wording regarding the purpose, goals, and resources available to the issues, but data is scarce. The COO has asked you to recommend what
team. to do first, as resources are not unlimited. Which of the following tools
would be most applicable to managing this?

‫التعليقات‬

These are essential elements of a good charter and must be clearly stated and understood Corrective Action/Preventative Action (CAPA)

.by the champion, team, and key stakeholders as part of the team's forming stage
Six Sigma

Pareto chart

Priorities matrix

* A SWOT analysis will force the company to look at their strengths and 5 ‫خيار‬

weaknesses. The best area to look for weaknesses is in:


6 ‫خيار‬

‫اإلجابة الصحيحة‬
Technical patents.
Priorities matrix
Competitive environment.

External environment.
‫التعليقات‬
Outdated technology.
A priorities matrix is ideal to create consensus scores on multiple alternatives using
whatever criteria your organization decides are important. This tool would be the best one

.to use at this point

https://docs.google.com/forms/d/e/1FAIpQLSe_EAdn9T3cnNgY1MBr4Yi0RS0utxLuIAYLclmRVChiUWq3TQ/viewscore?viewscore=AE0zAgCMmUqMl… 35/106 https://docs.google.com/forms/d/e/1FAIpQLSe_EAdn9T3cnNgY1MBr4Yi0RS0utxLuIAYLclmRVChiUWq3TQ/viewscore?viewscore=AE0zAgCMmUqMl… 36/106


‫ م‬10:39 2023/‫‏‬5/‫‏‬13 ASQ CMQ-OE : Practice Test #07 ‫ م‬10:39 2023/‫‏‬5/‫‏‬13 ASQ CMQ-OE : Practice Test #07

0/1* The quality and purchasing teams have identified potential risks. One 1/1* A manufacturing operation was examined in detail and one process step
risk is related to an opportunity to allow a supplier to develop a new was discovered to be incapable of meeting the daily production demand.
component that you initially started developing. What strategy would you Which of the following methodologies would best be used to improve the
use to manage this risk? process step?

Avoid Kaizen

Enhance Zero escapes

Accept Quality improvement

Share Theory of Constraints (TOC)

‫اإلجابة الصحيحة‬

Share

1/1 * Which of the following is an example of random sampling?

‫التعليقات‬
An inspector determines the location and characteristics of the sample group
Share is the strategy to pass ownership to a third party that might be better at managing

.the opportunity and maximizing the chance of it happening
An inspector inspects every 30th product

An inspector samples a portion of the population

An inspector selects a representative sample from the population


1/1* Two processes have the same value of Cp. Process one has a Cpk
value of 1.33 and process two has a Cpk value of 0.67. What can be said
about these processes?

1/1 * Looking at the following data:


The first is closer to being on center; the second is less centered
40, 20, 10, 20, 50, 90, 80, 30, 20, 40
The first is not capable; the second is
the mean is, the median is, and the mode is
Both processes are capable and centered

The first is in control; the second isn't 40, 20, 40

45, 70, 20

20, 40, 40

40, 35, 20

https://docs.google.com/forms/d/e/1FAIpQLSe_EAdn9T3cnNgY1MBr4Yi0RS0utxLuIAYLclmRVChiUWq3TQ/viewscore?viewscore=AE0zAgCMmUqMl… 37/106 https://docs.google.com/forms/d/e/1FAIpQLSe_EAdn9T3cnNgY1MBr4Yi0RS0utxLuIAYLclmRVChiUWq3TQ/viewscore?viewscore=AE0zAgCMmUqMl… 38/106


‫ م‬10:39 2023/‫‏‬5/‫‏‬13 ASQ CMQ-OE : Practice Test #07 ‫ م‬10:39 2023/‫‏‬5/‫‏‬13 ASQ CMQ-OE : Practice Test #07

0/1* Which of the following communication methods is an example of a type 0/1* Alpha Beta is a manufacturer of aircraft equipment and is a supplier to
of communication that needs to be established between the manufacturer eight of ten of the largest aircraft manufacturers in the world. Alpha Beta
and supplier? wants to identify which external customer group is most important before
they identify customer requirements and measure customer satisfaction.

Infrequent phone calls Which of the following should Alpha Beta consider their most important
external customer?
Occasional meetings

Non-emergency reporting procedures


Passengers who fly aboard aircraft built by Alpha Beta's customers
Routine meetings
Airlines who purchase aircraft built by Alpha Beta's customers
‫اإلجابة الصحيحة‬
Pilots who fly aircraft built by Alpha Beta's customers
Routine meetings
Employees who work for aircraft manufacturers

‫اإلجابة الصحيحة‬
‫التعليقات‬


.The most common type of communication occurs through routine meetings Employees who work for aircraft manufacturers

‫التعليقات‬

Aircraft manufacturers are Alpha Beta's most important direct external customers. Each
aircraft manufacturer has a number of employee stakeholders who are critical to Alpha
1/1* Without securing proper authorization, a project manager implements Beta. Examples include the customers they deal with from purchasing, quality,
project changes that add to the project's scope. The most likely impact to manufacturing and engineering. Each function has unique requirements and expectations
.of Alpha Beta
the project will be:

More project costs will be passed on to the stakeholders.

Project cost overruns will be a surprise to stakeholders when reported at the end of
the project.

Project shortcuts will be needed to meet deadlines.

The project exceeds its planned time and/or cost.

https://docs.google.com/forms/d/e/1FAIpQLSe_EAdn9T3cnNgY1MBr4Yi0RS0utxLuIAYLclmRVChiUWq3TQ/viewscore?viewscore=AE0zAgCMmUqMl… 39/106 https://docs.google.com/forms/d/e/1FAIpQLSe_EAdn9T3cnNgY1MBr4Yi0RS0utxLuIAYLclmRVChiUWq3TQ/viewscore?viewscore=AE0zAgCMmUqMl… 40/106


‫ م‬10:39 2023/‫‏‬5/‫‏‬13 ASQ CMQ-OE : Practice Test #07 ‫ م‬10:39 2023/‫‏‬5/‫‏‬13 ASQ CMQ-OE : Practice Test #07

0/1* As part of the annual business planning process, the Quality Manager
submitted a revised Quality Improvement (QI) plan-with a budget-to be
launched within the upcoming year. To improve the effectiveness of the 0/1* Green Chem (GC) is a chemical distribution company. After 10
total quality system, one al project that was approved is the purchase and consecutive years of increasing revenue and profits, GC unexpectedly
implementation of a Quality Improvement System (QIS) software lost several of its most valuable accounts, resulting in operating losses.
package to replace the current system, which contains numerous Follow-up interviews with lost customers revealed GC Account Managers
database areas and uncontrolled spreadsheets, reports, and other Ql- lacked the necessary experience and knowledge to support them in their
related documents. In addition, the current system is hard to search, industries. Top management realizes that they need to improve core
making it difficult for new improvement teams or auditors to retrieve customer retention and loyalty and help Account Managers become
specific information. experts in customer industries.

The Quality Manager agreed to be the project manager for the initiative. Top management wants to focus on the vital few (core customers) on
Her financial acumen and general management skills are essential for the whom GC's future depends. The useful many (non-core customers) are
project to succeed. It was estimated that the project would take 30 weeks still important, but GC must reallocate resources to meet and exceed the
from planning to project close. unique needs and expectations of core customers. Among GC's 12,000
customers, 10% account for 75% of all operating profits. The top 120
ROI estimated was a key reason for the project's approval. Two other key
customers (super core) each generate $50,000 or more in monthly
factors were that the new QIS would enable better internal and external
operating profit.
auditing and serve as a knowledge management system for new
improvement teams. For project ROI to be realized, what are some of the Which of the following is the most important first step in deploying
data that will need to be tracked, controlled, and more precisely profitability segmentation and will be most impactful in aligning service
estimated as the project is conducted? and delivery with customer needs?

GC must not ignore non-core customer needs and requirements


Auditor feedback after implementation, team labor hours, project resources
consumed, cost of the QIS, hours to maintain the system after implementation GC must emphasize growing sales volume and profitability of non-core customers

Team labor hours, project resources consumed, reduced cost of quality, new
GC must start with industry segments and then consider customer profitability
customer acquisitions after implementation
GC must determine the specific requirements of each core customer segment
User and auditor satisfaction with the new QIS, total project costs, reduction of
recurring quality issues
‫اإلجابة الصحيحة‬
Team labor hours, project resources consumed, cost of the QIS, reduced cost of
quality, cost to maintain the system after implementation GC must determine the specific requirements of each core customer segment

‫اإلجابة الصحيحة‬

Team labor hours, project resources consumed, cost of the QIS, reduced cost of ‫التعليقات‬
quality, cost to maintain the system after implementation GC needs to move beyond the old understanding of customer needs and expectations and
must determine how customer needs and requirements vary for each segment group. For
example, each Account Manager will be able to provide a great deal of special attention to
super core customers (lunch meetings, frequent follow-up, etc.) and provide more
‫التعليقات‬ .attention to deluxe core customers than non-core customers

All of these areas must be managed, controlled, and accurately estimated so that ROI can
be realized and reported by the end of the project that comes close to-or exceeds-the
.estimated ROI used to approve the project

https://docs.google.com/forms/d/e/1FAIpQLSe_EAdn9T3cnNgY1MBr4Yi0RS0utxLuIAYLclmRVChiUWq3TQ/viewscore?viewscore=AE0zAgCMmUqMl… 41/106 https://docs.google.com/forms/d/e/1FAIpQLSe_EAdn9T3cnNgY1MBr4Yi0RS0utxLuIAYLclmRVChiUWq3TQ/viewscore?viewscore=AE0zAgCMmUqMl… 42/106


‫ م‬10:39 2023/‫‏‬5/‫‏‬13 ASQ CMQ-OE : Practice Test #07 ‫ م‬10:39 2023/‫‏‬5/‫‏‬13 ASQ CMQ-OE : Practice Test #07

1/1* The inspection team was sent to a training/refresher course as part of a 1/1* Customer X has a new product line that will require coordination with its
personnel performance enhancement. Management would like to use this existing product lines manufactured by Company ABC. The executive
program as a selection process for sending the most skilled inspectors to management of Customer X sent Company ABC core information about
train business partners performing the same task. the new product line for review and input, due in 2 weeks. Which of the
following steps should Company ABC take to prepare the input requested
How would the training department help achieve management's goal? by the customer?

Help select the best inspector through a written test and focus on training the
Conduct a risk analysis associated with the feasible changes to internal processes
selected inspector

Create a training program that will help the chosen inspector relay knowledge to the Identify enhancements to internal process readiness for new product
business partner
Analyze current processes and establish action plans
Widen the scope of the refresher course to include training the trainer
Examine internal process capability versus the new product line's requirements
Create performance measurement criteria for pre-training and post-training

1/1* You have been appointed to lead a cross-functional team to solve an


ongoing problem that has existed for over a year. What is your focus in
your first team meeting?

Ensure that the team understands the problem and why each person is on the
team

Outline team management items including meeting times, locations, use of an


agenda, and distribution of minutes

Outline the nature of the problem to be solved and how it affects your department

Present a team meeting schedule to agree upon going forward

https://docs.google.com/forms/d/e/1FAIpQLSe_EAdn9T3cnNgY1MBr4Yi0RS0utxLuIAYLclmRVChiUWq3TQ/viewscore?viewscore=AE0zAgCMmUqMl… 43/106 https://docs.google.com/forms/d/e/1FAIpQLSe_EAdn9T3cnNgY1MBr4Yi0RS0utxLuIAYLclmRVChiUWq3TQ/viewscore?viewscore=AE0zAgCMmUqMl… 44/106


‫ م‬10:39 2023/‫‏‬5/‫‏‬13 ASQ CMQ-OE : Practice Test #07 ‫ م‬10:39 2023/‫‏‬5/‫‏‬13 ASQ CMQ-OE : Practice Test #07

0/1* Contract review is a critical part of a robust quality plan and has an 1/1 * A car dealer is:
important impact on the processes and financial performance of a
company. The DoRight Company seems to be successful in bidding for
A supplier to a car manufacturer.
high revenue customers but has a historical pattern of losing the account
after 6 months. Major issues reported by customers included late An internal customer of a car manufacturer.
deliveries and order processing errors. Customers tolerated issues during
the learning curve, but ultimately ended the contract as issues continued A subsidiary of a car manufacturer.
to persist after they formally complained. A kaizen team was assigned to
An external customer of a car manufacturer.
recommend an improvement plan for the contract review and fulfillment
process.

Which of the following would be a better plan for the team to consider?

1/1* You are an experienced training and development manager and have
A revised training program on how to address customer-specific requirements recently been hired by a not-for-profit hospital that has no formal
in addition to using default procedures on all orders employee quality training program. What would you recommend?
A customer-specific requirements documentation database to ensure accessibility
and common understanding of the requirements when developing plans to meet
Use before and after testing to measure understanding and retention
requirements
Focus on design, delivery, evaluation, behavior change, and results delivered
A customer survey program covering a 3- and 6-month progress check on new
accounts to respond to negative survey findings
Analyze customer needs, instructional design, content development, and pilot
A documented approach to deploying customer requirements with validation of testing
operational and quality control readiness
Align programs with the strategic and business needs of the hospital
‫اإلجابة الصحيحة‬

A documented approach to deploying customer requirements with validation of


operational and quality control readiness

‫التعليقات‬

This ensure that the process is well defined and that both operational and quality control
plans are in place and validated to ensure that specific customer requirements are being

.produced and released to the customer

https://docs.google.com/forms/d/e/1FAIpQLSe_EAdn9T3cnNgY1MBr4Yi0RS0utxLuIAYLclmRVChiUWq3TQ/viewscore?viewscore=AE0zAgCMmUqMl… 45/106 https://docs.google.com/forms/d/e/1FAIpQLSe_EAdn9T3cnNgY1MBr4Yi0RS0utxLuIAYLclmRVChiUWq3TQ/viewscore?viewscore=AE0zAgCMmUqMl… 46/106


‫ م‬10:39 2023/‫‏‬5/‫‏‬13 ASQ CMQ-OE : Practice Test #07 ‫ م‬10:39 2023/‫‏‬5/‫‏‬13 ASQ CMQ-OE : Practice Test #07

1/1* You are the Director of Quality and have recently worked with a third- 1/1* Which one of the following organizations is implementing strategic
party auditing group to pursue ISO registration. Due to the prevailing objectives as defined by Baldrige Criteria for Performance Excellence?
culture in your organization and the hard work you invested in designing
the quality management system, the organization did well on the audit.
City General Hospital publishes its statement of commitment to customer service
in the patient waiting room
One surprising non-conformance that was issued concerned how
documents within the quality manual were reviewed and approved. Since The Green County public school system submits its annual budget of expenses and
you lead the Quality Department-and because your supervisor (the COO) income to the school board for approval
delegated the quality manual and the quality management system to you-
Lamplight Testing Laboratories stores purchased reagents in alphabetical order for
you signed off on the revisions you introduced. As a result, you now need visibility into minimum order quantities at all 30 of its locations
to revise the process and the related procedure. That said, you are also
serving as the interim Director of Training, so you need to ensure that the The senior management of Burgers, Inc. restaurant introduces a plan to
members of the Quality Department know about and implement the increase market share by 10% within 2 years at its annual employee meeting
changes. The quality group consists of you and seven full-time-
employees (FTEs), some in your building and others working remotely.
The culture of the group consists of people who welcome personal and
professional challenges and master technology (and new concepts)
0/1 * Correcting deficiencies in job execution is not achieved by:
quickly. They tend to communicate internally via instant messaging.

Which of the following training delivery modes is optimal given this


Changing resources.
cultural summary?
Providing more objective feedback.

Traditional classroom lecture Adding or removing behavioral consequences.

Role-playing Training.

On-the-job training (OJT) ‫اإلجابة الصحيحة‬

Tech-based training Training.

‫التعليقات‬


.Training can do many things, but it cannot correct behavioral deficiencies

https://docs.google.com/forms/d/e/1FAIpQLSe_EAdn9T3cnNgY1MBr4Yi0RS0utxLuIAYLclmRVChiUWq3TQ/viewscore?viewscore=AE0zAgCMmUqMl… 47/106 https://docs.google.com/forms/d/e/1FAIpQLSe_EAdn9T3cnNgY1MBr4Yi0RS0utxLuIAYLclmRVChiUWq3TQ/viewscore?viewscore=AE0zAgCMmUqMl… 48/106


‫ م‬10:39 2023/‫‏‬5/‫‏‬13 ASQ CMQ-OE : Practice Test #07 ‫ م‬10:39 2023/‫‏‬5/‫‏‬13 ASQ CMQ-OE : Practice Test #07

1/1* Multiple departments compile performance metrics for strategic planning 0/1* You are an experienced training manager who has just been assigned to
meetings held quarterly at Oneida University. To maximize their a distribution center, the construction of which will be completed in
usefulness to Oneida management, the metrics should be: roughly 3 months. The goal is to have a trained workforce of about 50
people, drawn from the surrounding area, by the time the distribution
center is up and running. How would you proceed?
Aligned only with annual goals.

Determined based on historical organizational performance.


Ensure that the learning preferences of the individuals are properly reflected in
the training design
A key component of manager annual performance appraisals.
Determine what participants need to know and do as a result of the training
Aligned with strategic objectives.
Conduct a rigorous analysis of the return on training investment (ROTI)

Ensure top management commitment to training new hires

0/1* Which of the following supplier quality costs should be categorized as a ‫اإلجابة الصحيحة‬
prevention cost element?
Determine what participants need to know and do as a result of the training

Cost of disposition of nonconforming purchased materials

‫التعليقات‬
Cost of receiving and source inspection
Trying to design a training program without knowing the body of information that needs to
Cost of a site visit to correct a supplier service problem be addressed cannot be done; you need to have a detailed understanding of the job, its

.requirements, and any other content that needs to be known to be successful
Cost of supplier quality surveys

‫اإلجابة الصحيحة‬

Cost of supplier quality surveys


1/1 * The most beneficial use of project portfolio management is to:

‫التعليقات‬ Determine the best time to start a new project.

Supplier surveys are considered a prevention cost because they help prevent supplier Make room for a new project by canceling or postponing an existing project that

.problems will not produce as good a Return on Investment (ROI) as the new project.

Compare the net present value of a new project's ROI relative to continuing
previously approved projects.

Manage new and existing projects relative to strategic fit and the capacity of
required resources.

https://docs.google.com/forms/d/e/1FAIpQLSe_EAdn9T3cnNgY1MBr4Yi0RS0utxLuIAYLclmRVChiUWq3TQ/viewscore?viewscore=AE0zAgCMmUqMl… 49/106 https://docs.google.com/forms/d/e/1FAIpQLSe_EAdn9T3cnNgY1MBr4Yi0RS0utxLuIAYLclmRVChiUWq3TQ/viewscore?viewscore=AE0zAgCMmUqMl… 50/106


‫ م‬10:39 2023/‫‏‬5/‫‏‬13 ASQ CMQ-OE : Practice Test #07 ‫ م‬10:39 2023/‫‏‬5/‫‏‬13 ASQ CMQ-OE : Practice Test #07

0/1* A supplier is overwhelmed with the number of corrective action requests 0/1* The Quality Director sponsored a cross-functional team to perform a
issued by the former quality manager of Company ABC, resulting in continuous improvement project. As the project progressed, some
delayed responses. The new QA manager is reviewing the backlog of members dominated the team and caused resentment and conflicts with
open corrective actions. Which of the following would be an appropriate other team members. This has raised concerns for the Quality Director
action for the new QA manager to take with the supplier? that the project may be jeopardized. Which of the following steps would
be the most effective for the Quality Director to take?

Delete the open corrective actions from the system since they were requested by
the former quality manager Hold separate problem-solving meetings with each team member

Audit the supplier's facility to follow up on open items Hold an open forum with the team to discuss new ground rules, including equal
participation
Offer training to the supplier on the proper corrective action process
Help the team leader apply techniques to balance dialogue among all team
members
Work with the supplier to review and prioritize the open requests
Remove dominating members from the project to prevent further conflict
‫اإلجابة الصحيحة‬
‫اإلجابة الصحيحة‬
Work with the supplier to review and prioritize the open requests
Help the team leader apply techniques to balance dialogue among all team
members
‫التعليقات‬

Reviewing the open corrective actions is a good starting point and an opportunity to verify
similar occurrences. Prioritizing open corrective actions so the most important are ‫التعليقات‬

.addressed first helps both the supplier and the customer
This is a positive way to have all members provide their expertise and ideas to contribute
to the project. Team leaders and members sometimes need help from the sponsor or a

.skilled facilitator to overcome storming periods in their journey to a high performing team

https://docs.google.com/forms/d/e/1FAIpQLSe_EAdn9T3cnNgY1MBr4Yi0RS0utxLuIAYLclmRVChiUWq3TQ/viewscore?viewscore=AE0zAgCMmUqMl… 51/106 https://docs.google.com/forms/d/e/1FAIpQLSe_EAdn9T3cnNgY1MBr4Yi0RS0utxLuIAYLclmRVChiUWq3TQ/viewscore?viewscore=AE0zAgCMmUqMl… 52/106


‫ م‬10:39 2023/‫‏‬5/‫‏‬13 ASQ CMQ-OE : Practice Test #07 ‫ م‬10:39 2023/‫‏‬5/‫‏‬13 ASQ CMQ-OE : Practice Test #07

1/1* You are leading a team that keeps over-discussing issues and revisiting 1/1* A restaurant is having customer issues. Based on credit card purchases,
past agreements, so the team is falling behind schedule. You are the data shows an actual drop in return customer business (see balanced
frustrated with the effectiveness of team meetings, as are several of the scorecard results for prior 4 years, below). The restaurant is currently
team members. Which of the following options would be the most implementing ISO 9001 and needs to create a quality objective to
effective action for you to take? address this problem. What is the most important objective to address
this problem?

Invite a subject matter expert to join your next team meeting to expedite decisions

Invite a facilitator to join your next team meeting to help the team focus

Stress the importance of the project and the team's need to stay on track to the
team

Encourage the timekeeper to be more assertive in keeping the team on track

Increase satisfaction survey level to 100% of customers instead of using a sample


0/1* Which of the following proactive sources of customer information best size
help determine customer needs for developing new product offerings?
Improve food quality, i.e., make it worth the wait

Results from the most recently completed customer satisfaction survey Increase customer complaint resolution training to 100%

Complaint trends and lost customer analysis Decrease the average length of time to serve food to customers

Comments from sales representatives

Observation of customers and potential customers using current and related


products

‫اإلجابة الصحيحة‬

Observation of customers and potential customers using current and related


products

‫التعليقات‬

By observing actual and potential customers using your products and related competitive
products, you can evaluate ease of use, feature preferences, and affects associated with
.).use (pleased vs. frustrated, etc

https://docs.google.com/forms/d/e/1FAIpQLSe_EAdn9T3cnNgY1MBr4Yi0RS0utxLuIAYLclmRVChiUWq3TQ/viewscore?viewscore=AE0zAgCMmUqMl… 53/106 https://docs.google.com/forms/d/e/1FAIpQLSe_EAdn9T3cnNgY1MBr4Yi0RS0utxLuIAYLclmRVChiUWq3TQ/viewscore?viewscore=AE0zAgCMmUqMl… 54/106


‫ م‬10:39 2023/‫‏‬5/‫‏‬13 ASQ CMQ-OE : Practice Test #07 ‫ م‬10:39 2023/‫‏‬5/‫‏‬13 ASQ CMQ-OE : Practice Test #07

0/1* One of Oak Paper Company's plants has recurring failures for the same 1/1* Manufacturing is being moved to an offshore facility. How would you
customer process. Multi-functional teams have been sent to the plant to begin designing job- specific training for new assembly line workers?
investigate, and the team came up with a solution that was immediately
implemented. After a week, the same failure occurred despite the solution
Have new workers begin by assembling simulated product
implemented by the team.
Have an experienced operator provide on-the-job training
Increasing pressure from the customer forced management to send a
different team to evaluate the situation. Which of the following should this Develop a self-directed learning and apprentice program
new team recommend to management?
Define core competencies required for assembly line workers

Propose an improvement plan different from the solution proposed solution by the
first team

Establish metrics for the solution proposed by the first team


1/1* There are three shifts processing the same account. Recently, one of the

Analyze the results of the solution proposed by the first team shifts consisting of experienced workers has been producing consistent
errors for the last two consecutive weeks. The supervisor checked
Clarify the customer and organizational requirements internal operations variables like manpower, material, process, and
volume, and no changes were detected. The supervisor consulted with
‫اإلجابة الصحيحة‬
the Quality Director to help evaluate the issue and resolve the continuing
Clarify the customer and organizational requirements errors before a customer audit in 2 weeks.

Which of the following would the Quality Director take as the first step?

‫التعليقات‬
Modify processes as needed
One cannot solve the problem unless the problem is clearly defined. A good starting point
would be to evaluate if customer requirements are aligned with organizational capabilities
or requirements. If this alignment is not clear and fully understood by the system, failures Implement performance metric tracking

.will likely occur when trying to meet stakeholder expectations
Perform a gap assessment

Set improvement objectives and measurements

1/1 * Which of the following is an example of poka-yoke application?

Red zone on an automobile fuel indicator

Taking Tylenol tablets for headaches

Thorough testing before launching a product in the market

Password field provided on login pages of web applications

https://docs.google.com/forms/d/e/1FAIpQLSe_EAdn9T3cnNgY1MBr4Yi0RS0utxLuIAYLclmRVChiUWq3TQ/viewscore?viewscore=AE0zAgCMmUqMl… 55/106 https://docs.google.com/forms/d/e/1FAIpQLSe_EAdn9T3cnNgY1MBr4Yi0RS0utxLuIAYLclmRVChiUWq3TQ/viewscore?viewscore=AE0zAgCMmUqMl… 56/106


‫ م‬10:39 2023/‫‏‬5/‫‏‬13 ASQ CMQ-OE : Practice Test #07 ‫ م‬10:39 2023/‫‏‬5/‫‏‬13 ASQ CMQ-OE : Practice Test #07

0/1* When examining the process variation on a control chart for 0/1* The main purpose of the Malcolm Baldrige National Quality Award
improvement opportunities, the quality manger should:
(MBNQA) is to:

Decide how much of a reduction in variation should be established as a goal. Compete with the Deming Prize in Japan.

Consider that 80-95% of the variation are the result of workforce variation, and Replace the need for ISO 9001 registration.
therefore the responsibility of management.
Get manufacturers to take a process approach to improvement.
Focus improvement efforts on cycles or trends because these are opportunities
for improvements. Recognize organizations for business excellence that share best practices.

Identify the cause for the instances of the most extreme data points on the control
‫اإلجابة الصحيحة‬
chart and identify the root cause as an improvement opportunity.
Recognize organizations for business excellence that share best practices.
‫اإلجابة الصحيحة‬

Consider that 80-95% of the variation are the result of workforce variation, and
therefore the responsibility of management. ‫التعليقات‬

The Baldrige Award was established to address U.S. quality and productivity inadequacies
as compared to Japan. The award recognizes organizations in all sectors that have best
‫التعليقات‬ practices and produce results to a high degree of performance excellence. Recipients
share their practices and results to assist other organizations that are pursuing their own
Deming stated that process variation results from the workforce, but the responsibility lies .journey of excellence guided by the MBNQA criteria

.with management who developed and designed the process

https://docs.google.com/forms/d/e/1FAIpQLSe_EAdn9T3cnNgY1MBr4Yi0RS0utxLuIAYLclmRVChiUWq3TQ/viewscore?viewscore=AE0zAgCMmUqMl… 57/106 https://docs.google.com/forms/d/e/1FAIpQLSe_EAdn9T3cnNgY1MBr4Yi0RS0utxLuIAYLclmRVChiUWq3TQ/viewscore?viewscore=AE0zAgCMmUqMl… 58/106


‫ م‬10:39 2023/‫‏‬5/‫‏‬13 ASQ CMQ-OE : Practice Test #07 ‫ م‬10:39 2023/‫‏‬5/‫‏‬13 ASQ CMQ-OE : Practice Test #07

0/1* Your company has implemented a Lean Six Sigma project to improve the 1/1* An operational continuous improvement project resulted in projected net
volume of final customer shipments processed from your warehouse to savings, but after 6 months of implementation, a catastrophic failure
coincide with the October new sales promotion of your line of occurred. A Black Belt was assigned to review the data and write a report
snowboarding accessories. on lessons learned from this incident. What is the likely root cause the
Black Belt will find?
Which of the following should be done to measure the effectiveness of
this project?
The SIPOC model did not consider failure elements

Do nothing until the new year begins because the seasonal demand will have Root cause analysis was not performed
peaked and the sale promotion will have concluded
The Kano model was used to identify customer requirements
Record the shipment totals for October and ask management if they are satisfied
Metrics are not based on the balanced scorecard
Create shipment metrics to allow shifting variables and other changes in cyclical
demand during the season

Record actual shipments before and after the project

‫اإلجابة الصحيحة‬ 0/1* The new operations manager at a computer repair center has quickly
determined that production technicians are not productive and have low
Create shipment metrics to allow shifting variables and other changes in cyclical morale. What first step would the manager do to begin addressing this
demand during the season situation?

Implement a system of financial rewards for daily quotas


‫التعليقات‬
Gather more data and feedback to identify root cause and other constraints
Snowboarding is a seasonal product with a natural demand increase in the fall months.
This seasonal factor, along with the sales promotion, complicates the measurement
situation. Additional techniques must be applied to this complex situation to smooth the Retain an external consultant the new manager knows who has lean Six Sigma
.demand and sale promotion to accurately assess the lean six sigma project impact expertise

Improve the supervisor's leadership and critical thinking and problem-solving


skills

‫اإلجابة الصحيحة‬

Gather more data and feedback to identify root cause and other constraints

‫التعليقات‬

By collecting data and feedback the manager will better know what next steps are needed.
.Doing so demonstrates fact- based decision making, a key quality principle

https://docs.google.com/forms/d/e/1FAIpQLSe_EAdn9T3cnNgY1MBr4Yi0RS0utxLuIAYLclmRVChiUWq3TQ/viewscore?viewscore=AE0zAgCMmUqMl… 59/106 https://docs.google.com/forms/d/e/1FAIpQLSe_EAdn9T3cnNgY1MBr4Yi0RS0utxLuIAYLclmRVChiUWq3TQ/viewscore?viewscore=AE0zAgCMmUqMl… 60/106


‫ م‬10:39 2023/‫‏‬5/‫‏‬13 ASQ CMQ-OE : Practice Test #07 ‫ م‬10:39 2023/‫‏‬5/‫‏‬13 ASQ CMQ-OE : Practice Test #07

0/1* Which of the following sets of interested parties (or stakeholders) are 0/1* The ISO 9001 standard requires documented information for certain
most immediately served by a construction company's policy to enable its specific quality system requirements. Otherwise, having documented
employees to stop a project if a safety violation is observed? information like procedures, policies, work instructions, flowcharts or
records is based on:

Customers and suppliers Available resources and the size of the organization.

Suppliers and stakeholders Customer requirements.

Employees and stakeholders Registrar suggestions for process control.

Customers and employees Process effectiveness.

‫اإلجابة الصحيحة‬ ‫اإلجابة الصحيحة‬

Customers and employees Process effectiveness.

‫التعليقات‬ ‫التعليقات‬
Of the five types of interested parties- customers, stakeholders, employees, suppliers and Clause 7.5 of ISO 9001, Documented Information states that the need for additional
community-the two immediately served by the prevention of a safety concern would be documents is determined by the organization as necessary for ensuring the effectiveness
.)customers (product/ functional safety) and employees (self-safety
.of the quality system and its processes

1/1* The quality manager wishes to monitor the quality of incoming raw 1/1* All the following would be applicable to healthcare services quality
materials. If the quality can be measured on a continuous scale and is management except:
received infrequently, the appropriate control chart to use would be a:

National Committee for Quality Assurance (NCQA).


U-chart.
Joint Commission.
X-bar and R chart.
MBNQA.
P-chart.
AS 9100.
Individuals and Moving Range charts.

https://docs.google.com/forms/d/e/1FAIpQLSe_EAdn9T3cnNgY1MBr4Yi0RS0utxLuIAYLclmRVChiUWq3TQ/viewscore?viewscore=AE0zAgCMmUqMl… 61/106 https://docs.google.com/forms/d/e/1FAIpQLSe_EAdn9T3cnNgY1MBr4Yi0RS0utxLuIAYLclmRVChiUWq3TQ/viewscore?viewscore=AE0zAgCMmUqMl… 62/106


‫ م‬10:39 2023/‫‏‬5/‫‏‬13 ASQ CMQ-OE : Practice Test #07 ‫ م‬10:39 2023/‫‏‬5/‫‏‬13 ASQ CMQ-OE : Practice Test #07

0/1* A company deployed a blitz team method to produce quick results in its * Your team has identified a risk that is not controllable. What do you need to
new, fast-paced market sector. A unique characteristic of these types of develop to address this risk?
teams is that they have a full-time, dedicated:

Risk breakdown structure


Sponsor.
Fault tree analysis
Member.
FMEA
Facilitator.
Recovery activities
Leader.
‫اإلجابة الصحيحة‬
‫اإلجابة الصحيحة‬
Recovery activities
Facilitator.

‫التعليقات‬
‫التعليقات‬
For risks that are not controllable, recovery activities will need to be developed to deal with

.the effects of the risk
A facilitator is essential for blitz teams due to their short duration and the need to expedite
progress in three key areas: proper problem solving, data collection and analysis, and

.management of problematic group dynamics

1/1* A supplier has a recent history of late deliveries, defective products, and
violations of the contract. A supplier performance metric has been
developed by the customer and sent to the supplier to achieve
conformance with the terms of the contract for the deficiencies. Who
determines the frequency with which this supplier should be measured?

The customer's other suppliers

The supplier's quality department

A third-party auditor

The buyer

https://docs.google.com/forms/d/e/1FAIpQLSe_EAdn9T3cnNgY1MBr4Yi0RS0utxLuIAYLclmRVChiUWq3TQ/viewscore?viewscore=AE0zAgCMmUqMl… 63/106 https://docs.google.com/forms/d/e/1FAIpQLSe_EAdn9T3cnNgY1MBr4Yi0RS0utxLuIAYLclmRVChiUWq3TQ/viewscore?viewscore=AE0zAgCMmUqMl… 64/106


‫ م‬10:39 2023/‫‏‬5/‫‏‬13 ASQ CMQ-OE : Practice Test #07 ‫ م‬10:39 2023/‫‏‬5/‫‏‬13 ASQ CMQ-OE : Practice Test #07

1/1* A post-project review was performed after completing a 1-year project 1/1* A startup medical device repair service is in the process of formalizing
that established a new quality management system. To obtain and their organization design and organizational chart. The Quality Director
document the best possible information, the review should look at: researched options and will recommend the approach. She determined
that three strategic aspects are critical to the recommendation: agility of
internal resources, key customer focused teams, and ability to adjust
Flowcharts for the new quality management system and process. staffing levels to emerging or changing key customer demands. Which of
the following concepts would be best to recommend?
Final consultant costs for labor and resources needed to build the new quality
management system.
Product
Remaining opportunities for future improvement.
Geographic
Final reports on budget, deliverables, and the plan to manage the system.
Matrix

Traditional function

1/1 * Which statement best fits an optimized risk management process?

The plan is flexible enough to deal with all levels and kinds of risks

Risk mitigation plans include accurate and fixed documents

The cost of risk management equals the level of financial exposure

Overall costs associated with risk and their occurrence are minimized

1/1* The Director of Quality is concerned that some managers are granting
promotions to personnel who have lesser qualifications but have visible
rapport with the hiring manager. Others are saying that these promotions
show favoritism. As an ASQ member who believes in the Code of Ethics,
which action should the Director take to address this concern?

Encourage the managers to have good rapport with all personnel

Have the HR Director talk privately with the managers involved

Talk with the involved managers and explore their promotion criteria

Recommend an objective, system-wide approach to promotion to HR and top


management

https://docs.google.com/forms/d/e/1FAIpQLSe_EAdn9T3cnNgY1MBr4Yi0RS0utxLuIAYLclmRVChiUWq3TQ/viewscore?viewscore=AE0zAgCMmUqMl… 65/106 https://docs.google.com/forms/d/e/1FAIpQLSe_EAdn9T3cnNgY1MBr4Yi0RS0utxLuIAYLclmRVChiUWq3TQ/viewscore?viewscore=AE0zAgCMmUqMl… 66/106


‫ م‬10:39 2023/‫‏‬5/‫‏‬13 ASQ CMQ-OE : Practice Test #07 ‫ م‬10:39 2023/‫‏‬5/‫‏‬13 ASQ CMQ-OE : Practice Test #07

0/1* Company XYZ's improvement strategies include the customer feedback 0/1* You own a trucking company, and you've recently had problems in the
program as a tool to "hear" the VOC (voice of the customer) data. The delivery process for one of your clients. The loading at the client's factory
Director of Quality noted that customer feedback reported by sales goes smoothly, but once your trucks arrive at the client's warehouse, the
highlights internal failures as the possible cause of the customers' trucks typically wait 6-8 hours before workers unload the cargo. Every
feedback; the customer feedback reported by Account Managers minute that your trucks are parked and waiting costs your company
highlights potential customer- related issues. What would the Director of revenue.
Quality select as the most probable root cause of this difference in
feedback collection? Using process analysis to identify internal process-controlled variables,
you draw the conclusion that the trucks wait because:

The vision may need revision


The client's warehouse does not schedule enough employees to unload the cargo.
Objectives may require clarification to yield the desired return
The client's warehouse is too full to accept the cargo and must wait until their
Deployment of the initiative may not have been long enough to see trends trucks have loaded and left the factory.

Environmental information may require analysis Your trucking company needs more information from the client delaying the
unloading of the cargo.
‫اإلجابة الصحيحة‬
Your trucking company reassigned the dispatcher, and the client's warehouse was
Objectives may require clarification to yield the desired return not notified prior to the delivery.

‫اإلجابة الصحيحة‬

‫التعليقات‬ Your trucking company reassigned the dispatcher, and the client's warehouse was
not notified prior to the delivery.
Efforts/initiatives to support strategic planning must be based on the measurement of

.results against the expectations through organized, systemic feedback

‫التعليقات‬

Using process analysis in the form of flowcharting, 5 whys or Kaizen to perform root
cause analysis requires understanding how a system or process works, and the many
complex contributors, both technical and human, that can cause bottlenecks in the
system. The trucking company only has access to their own processes that could affect
the bottleneck. This answer refers to something in the trucking company's own processes

.and can be fixed

https://docs.google.com/forms/d/e/1FAIpQLSe_EAdn9T3cnNgY1MBr4Yi0RS0utxLuIAYLclmRVChiUWq3TQ/viewscore?viewscore=AE0zAgCMmUqMl… 67/106 https://docs.google.com/forms/d/e/1FAIpQLSe_EAdn9T3cnNgY1MBr4Yi0RS0utxLuIAYLclmRVChiUWq3TQ/viewscore?viewscore=AE0zAgCMmUqMl… 68/106


‫ م‬10:39 2023/‫‏‬5/‫‏‬13 ASQ CMQ-OE : Practice Test #07 ‫ م‬10:39 2023/‫‏‬5/‫‏‬13 ASQ CMQ-OE : Practice Test #07

0/1* In a global business environment, communication between organizations 0/1*   During an account status meeting, the account management team
must ensure that critical information is exchanged appropriately and is discovered gaps causing the misalignment of customer needs and
well understood. What should be done to achieve this objective? services offered to the customer. The results of implementing
transactional changes were not immediately visible to either the customer
or the service provider. To resolve the gaps and optimize lessons learned
Always include top management's policy and commitment to transparency with
any and all communications from the situation, which of the following tools should the account
management team implement for tracking the changes?
Make all actual changes that have been made to adapt communications to the
satisfaction of all parties visible to each organization
Matrix diagram
Identify existing differences in communicating practices and set up an agreement
O ahead of time on the protocol to be used when communicating between
Affinity diagram
organizations
Activity Network Diagram (AND)
Establish staff understanding of differences in culture, social practices, language
and business norms, and empower them to use protocols accordingly
Interrelationship digraph

‫اإلجابة الصحيحة‬
‫اإلجابة الصحيحة‬
Establish staff understanding of differences in culture, social practices, language
and business norms, and empower them to use protocols accordingly Interrelationship digraph

‫التعليقات‬ ‫التعليقات‬

Correct Establishing staff capability to assess global factors when communicating during An interrelationship digraph shows the cause- and-effect relationships between
daily work would be the best approach. It would deploy a globally sensitive system of components. Impact of the transactional changes will have more visibility to the team and
communications between organizations and guide the many ways communication takes
will help in the identification of
.place misalignment and early detection/prevention of gaps during the implementation of

.changes

1/1* Given a 4-week project involving 22 tasks and a budget of $22,000,


which project management tool would be better for displaying task
progress vs. time to project stakeholders?

Budget variance report

Work Breakdown Structure (WBS)

Activity Network Diagram (AND)

Gantt/milestone chart

https://docs.google.com/forms/d/e/1FAIpQLSe_EAdn9T3cnNgY1MBr4Yi0RS0utxLuIAYLclmRVChiUWq3TQ/viewscore?viewscore=AE0zAgCMmUqMl… 69/106 https://docs.google.com/forms/d/e/1FAIpQLSe_EAdn9T3cnNgY1MBr4Yi0RS0utxLuIAYLclmRVChiUWq3TQ/viewscore?viewscore=AE0zAgCMmUqMl… 70/106


‫ م‬10:39 2023/‫‏‬5/‫‏‬13 ASQ CMQ-OE : Practice Test #07 ‫ م‬10:39 2023/‫‏‬5/‫‏‬13 ASQ CMQ-OE : Practice Test #07

0/1* Bobby is the Production Manager at No Sock Holes, Inc. Her roles and 1/1* We usually describe a product or service in terms of several dimensions
responsibilities as Production Manager vary on a daily basis. For or characteristics. After receiving a service, we might describe the service
example
provider as fast. Which of the following aspects relates to the comment,
"the service provider is fast?"
. On Monday, Bobby reviewed the strategic plan to determine how to
schedule future production to meet identified long-term objectives.
Responsiveness
• On Tuesday, Bobby met with the largest customer to discuss quality
issues with fabric wear and to gain agreement on a resolution. Completeness

. On Wednesday, Bobby shared the fabric wear resolutions with Professionalism


production supervisors hoping to remedy the issue and maintain a
positive relationship with the customer. Availability

• On Thursday, Bobby met with a group of interns to address any


questions they had regarding their roles in the Production Department.

On which day did Bobby demonstrate the role of conflict resolution? 1/1* Joe, a team leader at an aircraft supply company, is interviewing
candidates to fill a position for a key project. Joe is eager to ensure that
Tuesday the right participant is selected. What model will best help Joe determine
the right candidate?
Wednesday

Monday KESAA

Thursday Myers-Briggs Type Indicator (MBTI)

‫اإلجابة الصحيحة‬ DiSC

Tuesday Maslow's hierarchy of needs

‫التعليقات‬

Meeting with the customer to identify concerns and gain a resolution is an example of

.principled negotiation, also known as interest- based conflict resolution

https://docs.google.com/forms/d/e/1FAIpQLSe_EAdn9T3cnNgY1MBr4Yi0RS0utxLuIAYLclmRVChiUWq3TQ/viewscore?viewscore=AE0zAgCMmUqMl… 71/106 https://docs.google.com/forms/d/e/1FAIpQLSe_EAdn9T3cnNgY1MBr4Yi0RS0utxLuIAYLclmRVChiUWq3TQ/viewscore?viewscore=AE0zAgCMmUqMl… 72/106


‫ م‬10:39 2023/‫‏‬5/‫‏‬13 ASQ CMQ-OE : Practice Test #07 ‫ م‬10:39 2023/‫‏‬5/‫‏‬13 ASQ CMQ-OE : Practice Test #07

1/1* You are a plant manager who must lay off 10% of your second shift 1/1* Automating one step in the production process generally improves
workforce due to a market slowdown. How would you communicate the productivity and efficiency for this step in the process. However, the entire
pending layoff? process realizes constrained performance due to process limitations
located at other steps in the process. By not evaluating the constraints of
the entire process, the implementation of the automated process step's
Post an announcement on all bulletin boards
impact on the process was:
Have the Human Resources Department communicate the decision to all
employees
Process fault tolerance.
Send an email to all shift supervisors and request that they inform their employees
within 24 hours Results-oriented management.

Starting with the second shift workforce, hold all-employee meetings so Management by exception.
Oleadership can present the situation and how the company will be conducting
the layoff process Sub-optimization.

1/1* A bookseller sells entirely through a website. Which of the following


would best help the bookseller manage customer diversity?

Acquiring a new server to host the website

Working with customers to obtain discount pricing

Upgrading software to utilize cloud data storage and high-speed access

Developing a new multilingual home page with custom offerings

https://docs.google.com/forms/d/e/1FAIpQLSe_EAdn9T3cnNgY1MBr4Yi0RS0utxLuIAYLclmRVChiUWq3TQ/viewscore?viewscore=AE0zAgCMmUqMl… 73/106 https://docs.google.com/forms/d/e/1FAIpQLSe_EAdn9T3cnNgY1MBr4Yi0RS0utxLuIAYLclmRVChiUWq3TQ/viewscore?viewscore=AE0zAgCMmUqMl… 74/106


‫ م‬10:39 2023/‫‏‬5/‫‏‬13 ASQ CMQ-OE : Practice Test #07 ‫ م‬10:39 2023/‫‏‬5/‫‏‬13 ASQ CMQ-OE : Practice Test #07

1/1* You have just started as Quality Director at a manufacturing firm with * Company XYZ is in the process of restructuring its knowledge management
multiple locations, with 1500 employees total. You have found that program to prepare for an acquisition that will extend market territory and
procedural documentation is lacking and if there is any, it has not been product lines. Which of the following would help support an effective
updated in a long time. There are known issues with too much internal restructuring period?
rework and a lack of planning resources and efforts. Your short list has 25
possible areas to fix. The company COO has provided only a general
direction to increase capacity. Not all managers see the same issues, and Promote more individual software privileges to allow staff to design personal areas
in the knowledge management system
disagreement is common between departments and up and down the
management chain. Promote authoring identification information to ensure credit is given to deserving
individuals that add content to the site
You have a large budget to work with as a large potential customer has
been identified, if you can meet their stringent cost and quality needs. Focus on IT and database support to ensure effective knowledge management
interface and help desk response
The company does not use the usual quality tools, and you were hired to
help. Specify benefits and protocols for sharing knowledge in the database and
encourage a culture of shifting tacit knowledge to explicit knowledge
Your suppliers are now delivering your components for your new
innovative product. What do you need to do to manage the risks
associated with the new suppliers?

1/1* A fast food chain wants to evaluate its meat suppliers' controls for
Conduct supplier quality issue reviews product integrity and effectiveness of product recall procedures. Which of
the following is the best action to accomplish this purpose?
Encourage suppliers to do fault tree analysis

Provide training in general manufacturing concepts Request to review records of previous product recalls the company has
experienced
Perform supplier surveillance
Make the supplier provide the names and qualifications of their product recall team

Conduct a mock recall of a given lot of meat and expect 100% traceability

1/1 * Which of the following is a function of the 5 Whys methodology? Contract a third-party auditor to review the supplier's product recall procedure

To show whether there is correlation between two variables

To determine all the possible causes for an identified event

To visually display the sequence of steps in a process

To determine the root cause for an identified problem

https://docs.google.com/forms/d/e/1FAIpQLSe_EAdn9T3cnNgY1MBr4Yi0RS0utxLuIAYLclmRVChiUWq3TQ/viewscore?viewscore=AE0zAgCMmUqMl… 75/106 https://docs.google.com/forms/d/e/1FAIpQLSe_EAdn9T3cnNgY1MBr4Yi0RS0utxLuIAYLclmRVChiUWq3TQ/viewscore?viewscore=AE0zAgCMmUqMl… 76/106


‫ م‬10:40 2023/‫‏‬5/‫‏‬13 ASQ CMQ-OE : Practice Test #07 ‫ م‬10:40 2023/‫‏‬5/‫‏‬13 ASQ CMQ-OE : Practice Test #07

0/1* The training department is preparing to update a training program for 1/1* Central Hospital began its first strategic operations planning process to
new hires. The quality director suggested the inclusion of lean principles ensure its long-term success. Before implementing action plans to
in the training material so the new employees will be more prepared to achieve the defined strategic objectives, Central Hospital reviews the
embrace the lean culture once assigned to their posts. Using the phases master means matrix to:
of the instructional design ADDIE model, where would the lean principles
first be addressed in detail in the model?
Assign management responsible for each objective.

Provide a statistical justification for the objectives.


Implementation phase

Estimate the next annual budget.


Evaluation phase

Identify potential resource conflicts.


Analysis phase

5 ‫خيار‬
Design phase

6 ‫خيار‬
‫اإلجابة الصحيحة‬

Design phase

‫التعليقات‬

Within the design phase, practical decisions are made regarding course content, delivery
methods, measurement and evaluation, and implementation strategies. A mention of lean
principles would first occur within the analysis phase but addressing the principles in

.detail occurs in the design phase

https://docs.google.com/forms/d/e/1FAIpQLSe_EAdn9T3cnNgY1MBr4Yi0RS0utxLuIAYLclmRVChiUWq3TQ/viewscore?viewscore=AE0zAgCMmUqMl… 77/106 https://docs.google.com/forms/d/e/1FAIpQLSe_EAdn9T3cnNgY1MBr4Yi0RS0utxLuIAYLclmRVChiUWq3TQ/viewscore?viewscore=AE0zAgCMmUqMl… 78/106


‫ م‬10:40 2023/‫‏‬5/‫‏‬13 ASQ CMQ-OE : Practice Test #07 ‫ م‬10:40 2023/‫‏‬5/‫‏‬13 ASQ CMQ-OE : Practice Test #07

0/1* You've just spent the day working the counter with your customer service 0/1* The strategy planning team is in the middle of assessing the best
team to understand their issues and identify opportunities to improve improvement initiative for Process X. What type of data collection would
customer service. There were some minor events that could have been the team likely choose to drive this program?
handled better, but most everyone seemed to be doing the right thing.
Team members were especially polite when handling 'difficult' customers.
Competitive analysis of the market
Which of the following actions should you take to improve customer
service? Regulatory trends governing the process

Core competencies of specific competitors with same process


Determine root cause and corrective actions for "difficult' customers
Benchmarking data collection
Implement a training program for conflict resolution techniques
‫اإلجابة الصحيحة‬
Document the minor events for future improvement projects
Benchmarking data collection
Acknowledge the good service at a staff meeting

‫اإلجابة الصحيحة‬
‫التعليقات‬
Acknowledge the good service at a staff meeting
Benchmarking provides input on the performance requirement for the process to assess

.compatibility with strategic planning's objectives

‫التعليقات‬

When possible, commend employees for their excellent service to customers. Praising
employees raises morale and increases the likelihood that they will continue to provide

.this type of exceptional service in the future 1/1* Which of the following principles is most important to achieve excellent
customer service?

Remember that "the customer is always right"

Make the customer realize you are not responsible for the problem

Strictly adhere to company policy for handling problems

Treat the customer with courtesy, politeness, and respect

https://docs.google.com/forms/d/e/1FAIpQLSe_EAdn9T3cnNgY1MBr4Yi0RS0utxLuIAYLclmRVChiUWq3TQ/viewscore?viewscore=AE0zAgCMmUqMl… 79/106 https://docs.google.com/forms/d/e/1FAIpQLSe_EAdn9T3cnNgY1MBr4Yi0RS0utxLuIAYLclmRVChiUWq3TQ/viewscore?viewscore=AE0zAgCMmUqMl… 80/106


‫ م‬10:40 2023/‫‏‬5/‫‏‬13 ASQ CMQ-OE : Practice Test #07 ‫ م‬10:40 2023/‫‏‬5/‫‏‬13 ASQ CMQ-OE : Practice Test #07

1/1* One of the most critical elements necessary when establishing a 1/1 * Which of the following is a good listening skill at work?
customer-supplier partnership begins with:

Bring the subject back to your point so that the communication does not go off-
Asking for reference contacts from their supplier's larger and longer contract track
customers
Make the best use of time by shaping your response so there are no long pauses
Requiring specific information from suppliers related to their technical resources,
sales forecasts, and best practices Multitask while the other person is talking to stay productive

Having a strategy for having as many suppliers as possible As communication progresses, offer a summary of what you think you heard to
the person speaking
Developing an honest and open communications process between the
customer and suppliers

0/1* Which of the following actions is needed to create employment stability


when improvement projects result in increased productivity?
1/1* An airline wants to focus on core customers with high current or future
value, i.e., customers who frequently access flight services. Using a
frequent flier program, airlines offer a higher level of service (such as Downsize staff prior to improvement activities being conducted
early boarding privileges) to their frequent flyers, while still meeting the
Physically separate employees to have them focus on their job tasks and get more
needs of their other passengers. To maximize customer satisfaction in
done
segments vital to the organization's well-being, the airline should:
Address potential staff reassignments or reductions prior to launching
improvement activities
Enter into partnerships or other forms of strategic alliances.
Communicate information that is required for their new job methods
Evaluate the cost of service versus the economic benefit.
‫اإلجابة الصحيحة‬
Develop a strategy to reach appropriate segments of potential customers.
Address potential staff reassignments or reductions prior to launching improvement
Determine the needs and expectations of core customer groups. activities

‫التعليقات‬

Threats to individual jobs as a result of improvement is a strategic element an


.organization must address prior to conducting improvement activities

https://docs.google.com/forms/d/e/1FAIpQLSe_EAdn9T3cnNgY1MBr4Yi0RS0utxLuIAYLclmRVChiUWq3TQ/viewscore?viewscore=AE0zAgCMmUqMl… 81/106 https://docs.google.com/forms/d/e/1FAIpQLSe_EAdn9T3cnNgY1MBr4Yi0RS0utxLuIAYLclmRVChiUWq3TQ/viewscore?viewscore=AE0zAgCMmUqMl… 82/106


‫ م‬10:40 2023/‫‏‬5/‫‏‬13 ASQ CMQ-OE : Practice Test #07 ‫ م‬10:40 2023/‫‏‬5/‫‏‬13 ASQ CMQ-OE : Practice Test #07

1/1* Tacos Everywhere tracks performance characteristics on their 25 1/1* One of Dr. Deming's 14 points for management is to 'cease dependence
vendors. Each characteristic is scored on a scale of one to five and then upon inspection." Which of the following is the underlying tenet of this
shared with the vendor. In which stage of the supply chain would the statement?
process of sharing individual performance information with a vendor
occur?
100% inspection can miss as much as 10% or more of defects

Workers should inspect their own work


Benchmarking

Companies employ too many inspectors


Desk audit

Quality must be built in by the process


Complaint handling

Post-award surveillance

1/1*  The use of QCI's CMQ/OE study materials to pass ASQ's certification
exam is most typical of what training effect measurement level?
1/1* Your company is a well-established supplier of wiring components for
lighting manufacturers. A condition of selling to a new customer is ISO
Reaction
certification, which you believe will have benefits for all customers, not
just the new one. How would you launch the training component for the Learning
ISO certification process?
Behavior

Determine the ISO registrar with which you want to work Results

Identify a training company that can do the training

Use a mix of classroom, on-the-job, and computer-based learning

Determine gaps between current training practices and records and ISO
requirements

https://docs.google.com/forms/d/e/1FAIpQLSe_EAdn9T3cnNgY1MBr4Yi0RS0utxLuIAYLclmRVChiUWq3TQ/viewscore?viewscore=AE0zAgCMmUqMl… 83/106 https://docs.google.com/forms/d/e/1FAIpQLSe_EAdn9T3cnNgY1MBr4Yi0RS0utxLuIAYLclmRVChiUWq3TQ/viewscore?viewscore=AE0zAgCMmUqMl… 84/106


‫ م‬10:40 2023/‫‏‬5/‫‏‬13 ASQ CMQ-OE : Practice Test #07 ‫ م‬10:40 2023/‫‏‬5/‫‏‬13 ASQ CMQ-OE : Practice Test #07

1/1* As a key organizational leader with prior experience championing teams, 1/1* The Director of Quality was part of the team tasked to evaluate the root
you have been tasked with implementing a change management cause(s) of a significant increase in overtime by one of its centers located
approach for the organization. The organization's core business is at Laredo, Tex. for the last 6 months. Other increasing variables noted
providing call center services to companies in the U.S. and Canada. Last are customer complaints and rate of internal error reported by the quality
year, the organization began marketing and sales efforts to acquire inspector stationed at Laredo. The company's remaining 19 centers
clients in Europe and China, and three new call centers have opened as offering the same business service seem to be performing well in
a result with more to follow. comparison with Laredo.

Top management wants the change management approach to Which of the following actions would the Director of Quality recommend
accomplish the following: as the initial step for evaluation by the team?

1. Transition all call centers to an empowered, natural work team


structure.
Consider outsourcing the Laredo tasks to maximize productivity

2. Identify and develop change agents within the call center workforce
Perform a time study of the Laredo center's tasks
that will drive continual improvement. 3. Establish buy-in on performance
metrics that will drive center efficiency and a high-degree of caller Verify the Laredo center's compliance with procedures
satisfaction. You are given the resources to hire an external consultant as
a change agent for the initiative. Verify the participation of supervisors in the execution of daily tasks

A known obstacle to change that must be addressed by this initiative is


how well empowerment will be embraced by staff in the new centers
given the cultural differences between the new centers and those in the
U.S. and Canada.

Which of the following evaluation tools would be more applicable to


managing this aspect of change?

Knowledge, Experience, Skills, Aptitude, and Attitude (KESAA)

Maslow's Hierarchy of Needs

Gap analysis of current to future state

Force Field Analysis (FFA)

https://docs.google.com/forms/d/e/1FAIpQLSe_EAdn9T3cnNgY1MBr4Yi0RS0utxLuIAYLclmRVChiUWq3TQ/viewscore?viewscore=AE0zAgCMmUqMl… 85/106 https://docs.google.com/forms/d/e/1FAIpQLSe_EAdn9T3cnNgY1MBr4Yi0RS0utxLuIAYLclmRVChiUWq3TQ/viewscore?viewscore=AE0zAgCMmUqMl… 86/106


‫ م‬10:40 2023/‫‏‬5/‫‏‬13 ASQ CMQ-OE : Practice Test #07 ‫ م‬10:40 2023/‫‏‬5/‫‏‬13 ASQ CMQ-OE : Practice Test #07

0/1* Teams can be ineffective at problem solving despite having a committed 0/1 * Which of the following is a risk associated with the purchasing process?
group of process experts amongst them. What is the more likely reason
for this?
Technical requirements in purchase orders are explained to a supplier by a non-
purchasing employee
The team is not properly trained and monitored
A supplier's quality system lacks its own internal auditing team
The team is not documenting progress in meeting minutes
A supplier also works with a company competitor
The team's focus is on breakthrough improvement rather than incremental
improvement A supplier starts working with the company before it is formally approved

The team is not rewarded for achieving milestone accomplishments ‫اإلجابة الصحيحة‬

‫اإلجابة الصحيحة‬
A supplier starts working with the company before it is formally approved

The team is not properly trained and monitored

‫التعليقات‬

Purchasing personnel and supplier quality engineers are responsible for approving
‫التعليقات‬ suppliers for use. When others can bypass purchasing and initiate working relationships
with suppliers they prefer, it creates risk to the purchasing process and jeopardizes proper
Teams must be prepared and trained to perform the tasks of the team, which in this case .supplier management
includes problem solving and subsequent solution development. Various quality tools like
Plan-Do- Check-Act (PDCA) and DMAIC and lean and Six Sigma skills benefit a team's
problem-solving ability. Also, this type of issue can be caught early if a team is properly

.monitored by the sponsor or a facilitator

1/1* You have just been hired as the quality manager at a traditional, mid-
sized manufacturing company. Historically, training has been viewed by
senior management as a necessary expense to be minimized, not as an
1/1* Which term represents the variation between people using the same investment in people.
equipment?
How would you change this thinking?

Reliability
Use post-training surveys to demonstrate the extent to which trainees believe they
benefit from the training
Reproducibility
Initiate lean principles at all levels throughout the company
Repeatability
Calculate training expenditures as a percent of expenses and show that your
Accuracy expense is below the industry average

Implement a pilot training program and show the impact on the bottom line

https://docs.google.com/forms/d/e/1FAIpQLSe_EAdn9T3cnNgY1MBr4Yi0RS0utxLuIAYLclmRVChiUWq3TQ/viewscore?viewscore=AE0zAgCMmUqMl… 87/106 https://docs.google.com/forms/d/e/1FAIpQLSe_EAdn9T3cnNgY1MBr4Yi0RS0utxLuIAYLclmRVChiUWq3TQ/viewscore?viewscore=AE0zAgCMmUqMl… 88/106


‫ م‬10:40 2023/‫‏‬5/‫‏‬13 ASQ CMQ-OE : Practice Test #07 ‫ م‬10:40 2023/‫‏‬5/‫‏‬13 ASQ CMQ-OE : Practice Test #07

1/1* An operator is to assemble certain parts without error. Each assembly 1/1* As a quality manager, you need to provide awareness of your company's
may be different depending on the customer order. Which of the following quality philosophy to new employees, including topics on why quality is
is the best example of error-proofing for this situation? important, what quality means in our environment, how quality affects our
daily work, and where we can begin to apply quality concepts and
techniques.

Each operator is trained on which parts go into each assembly being made How would you provide awareness of your company's quality philosophy
available to customers
to new employees?
When the operator is to put together an assembly, they are given a list containing
only the parts for that assembly

Standardized work instructions are written and posted at the operations; these Job-specific technical training
contain instructions for all the assemblies
Visual management boards displaying quality related metrics
When the operator is to put together an assembly, only the bins containing the
parts for that assembly light up Quality public relations strategies

Dedicated quality training session(s) within the new hire orientation process

1/1* The Green Belt program was created as part of a strategy by a logistics
company's CEO of saving one million dollars by next year. The sales
department was tasked to 'sell' the continuous improvement service to
customers. After 9 months of trying to solicit interested customers, the
program was a failure and did not bring the anticipated revenue. What
would be the likely lesson that the management team learned from this
situation?

The mission statement needs to be frequently revised to keep it dynamic and


continually effective

There should be a greater emphasis on the tactical and operational plans

The action plan was clear and should have been deployed effectively

External forces (e.g., customer requirements) must be part of strategic planning

https://docs.google.com/forms/d/e/1FAIpQLSe_EAdn9T3cnNgY1MBr4Yi0RS0utxLuIAYLclmRVChiUWq3TQ/viewscore?viewscore=AE0zAgCMmUqMl… 89/106 https://docs.google.com/forms/d/e/1FAIpQLSe_EAdn9T3cnNgY1MBr4Yi0RS0utxLuIAYLclmRVChiUWq3TQ/viewscore?viewscore=AE0zAgCMmUqMl… 90/106


‫ م‬10:40 2023/‫‏‬5/‫‏‬13 ASQ CMQ-OE : Practice Test #07 ‫ م‬10:40 2023/‫‏‬5/‫‏‬13 ASQ CMQ-OE : Practice Test #07

0/1* You are a plant manager and have concluded that improved plant 1/1* The assembly of Product A consists of multiple sub-assembly lines. The
performance will depend on meeting recently set process goals. The process has been stable for the last few years. No major issues were
downtime goal is not being met. What should you do? encountered until the last few days; there is now a high reject rate at the
last sub-assembly line. The quality engineer assigned to this sub-
assembly line immediately begins an investigation.
Develop process objectives to support the process goal
Which of the following should the quality engineer consider as the
Call a meeting with the operations and maintenance managers to stress the
importance of meeting process goals simplest starting step?

Post actual and targeted downtime information on the shop floor so operators will
see the performance gap and can make suggestions for improvement Reassess the process objectives and acceptance measurements

Investigate which of the process objectives is not being met and take corrective Review performance metric history for all sub-assembly lines
action
Review raw material specification and contact the supplier
5 ‫خيار‬
Identify any change in internal customer interfaces
6 ‫خيار‬

7 ‫خيار‬

‫اإلجابة الصحيحة‬
1/1* There are three steps needed to manufacture the product in the order of
Investigate which of the process objectives is not being met and take corrective A, B, C. The diagram below shows operator time and machine time for
action each step in a single-piece flow process and only one operator for each
step, but multiple machines are available for all steps.

‫التعليقات‬

Process objectives are the intended actions needed if process goals are to be met. Finding
out which actions are driving the problem will allow concrete and specific manager

.actions to correct the problem of missing the downtime goal

Improve step C operator time

Improve step C machine time

Improve step B machine time

Improve step B operator time

https://docs.google.com/forms/d/e/1FAIpQLSe_EAdn9T3cnNgY1MBr4Yi0RS0utxLuIAYLclmRVChiUWq3TQ/viewscore?viewscore=AE0zAgCMmUqMl… 91/106 https://docs.google.com/forms/d/e/1FAIpQLSe_EAdn9T3cnNgY1MBr4Yi0RS0utxLuIAYLclmRVChiUWq3TQ/viewscore?viewscore=AE0zAgCMmUqMl… 92/106


‫ م‬10:40 2023/‫‏‬5/‫‏‬13 ASQ CMQ-OE : Practice Test #07 ‫ م‬10:40 2023/‫‏‬5/‫‏‬13 ASQ CMQ-OE : Practice Test #07

0/1* When developing a supplier quality plan that ensures compliance with 0/1* To make the case to top management, the best reason for categorizing
government contracting requirements, the quality manager should: the cost of quality into failure costs, appraisal costs, and prevention costs
is to be able to:

Conduct supplier surveys at regular intervals to determine that levels of quality


are maintained throughout the supplier's company. Decrease appraisal costs without impacting product quality.

Establish a specific schedule for quality management system audits using the ISO Justify a budget increase for prevention efforts.
9001 requirements.
Reduce the total cost of quality to as near zero as possible.
Develop a plan for ongoing source inspections to ensure that schedules and levels
of quality are maintained.
Align quality goals with business objectives.

Include multiple methods for ensuring compliance with quality requirements.


‫اإلجابة الصحيحة‬

‫اإلجابة الصحيحة‬ Align quality goals with business objectives.

Include multiple methods for ensuring compliance with quality requirements.

‫التعليقات‬

‫التعليقات‬ COQ programs help translate quality improvements into language top management can

.understand and demonstrates the positive impact of any quality improvement programs
Multiple methods of gathering data from a supplier provides assurance that the company

.is meeting contract requirements

1/1* Team development progresses when team members have the basic

1/1* A quality manager needs to determine whether all aspects of a supplier's


skills to:
quality program are functioning as intended. The most appropriate audit
for this purpose is the: Manage a project.

Tolerate difficult team situations.


Process audit.
Apply Plan-Do-Check-Act (PDCA) to problem solving.
Third party audit.
Address storming by norming.
Product audit.

Systems audit.

https://docs.google.com/forms/d/e/1FAIpQLSe_EAdn9T3cnNgY1MBr4Yi0RS0utxLuIAYLclmRVChiUWq3TQ/viewscore?viewscore=AE0zAgCMmUqMl… 93/106 https://docs.google.com/forms/d/e/1FAIpQLSe_EAdn9T3cnNgY1MBr4Yi0RS0utxLuIAYLclmRVChiUWq3TQ/viewscore?viewscore=AE0zAgCMmUqMl… 94/106


‫ م‬10:40 2023/‫‏‬5/‫‏‬13 ASQ CMQ-OE : Practice Test #07 ‫ م‬10:40 2023/‫‏‬5/‫‏‬13 ASQ CMQ-OE : Practice Test #07

0/1* The design of an information management system should first consider 0/1* Your distribution center has experienced an increase in customer order
which of these fundamental questions: volume. This increase is attributed to the warehouse team's recent efforts
to make significant improvements in their order fulfillment and shipping
process. Two new employees have been hired to join this warehouse
How does the system interface with remote access networks and the internet?
team. The team is now in the:
How should standard reports be formatted and accessed?

Norming stage, because the team has already established agreements.


What information and data is needed and by whom?

Who is responsible for the accuracy and control of the data and information Forming stage, because it has new members and will need to rebuild the team.
entered?
Storming stage, because the team members could quickly encounter issues or
conflicts with the new employees.
‫اإلجابة الصحيحة‬

What information and data is needed and by whom? Performing stage, because this team has already delivered improved results.

‫اإلجابة الصحيحة‬

‫التعليقات‬ Forming stage, because it has new members and will need to rebuild the team.

Determining what information is required and who needs the information is a basic
question to answer when establishing the overall design of an information management
.system. Answering this question will drive how the other questions are answered ‫التعليقات‬

This is now a new team due to the addition of two new team members. The team building
process must begin from the forming stage and work itself back up to the performing
.stage, storming and norming along the way

1/1* Evidence that the project objectives and scope have been defined and
approved by management would be found in which of the following
documents?

Project schedule

Lessons learned report

Status reports

Project charter

https://docs.google.com/forms/d/e/1FAIpQLSe_EAdn9T3cnNgY1MBr4Yi0RS0utxLuIAYLclmRVChiUWq3TQ/viewscore?viewscore=AE0zAgCMmUqMl… 95/106 https://docs.google.com/forms/d/e/1FAIpQLSe_EAdn9T3cnNgY1MBr4Yi0RS0utxLuIAYLclmRVChiUWq3TQ/viewscore?viewscore=AE0zAgCMmUqMl… 96/106


‫ م‬10:40 2023/‫‏‬5/‫‏‬13 ASQ CMQ-OE : Practice Test #07 ‫ م‬10:40 2023/‫‏‬5/‫‏‬13 ASQ CMQ-OE : Practice Test #07

0/1* Situational leadership methods have four managing styles and the 1/1* Your task is to evaluate capital budget proposals and make
specific situation and skill level of the employee determine which style to recommendations to senior management. Which of the following well-
use. For this approach, which order of styles best helps the employee accepted methods would you use?
mature?

Provide each department with the appropriate forms to complete, together with
examples of a properly completed budget proposal
Involving, participating, empowering, directing
Rewrite proposals that do not conform to the organization's capital budget
Monitoring, directing, delegating, empowering guidelines

Planning, doing, checking, acting For discretionary expenditures, rank projects based on their projected rate of return
and reject those that did not meet the cutoff
Directing, selling, participating, delegating
Use SMART guidelines in combination with alignment with strategic objectives
‫اإلجابة الصحيحة‬

Directing, selling, participating, delegating

‫التعليقات‬

These are the four styles and their order of use as described by Hersey & Blanchard:
directing (for new hires or inexperienced people); selling (for motivated people who lack
maturity or ability but can do more of the task), participating (for people who are able to
do most of the task and may need occasional help), and delegating (for people who are

.)fully competent, know what to do, and are motivated to do it

1/1* Organizational success is largely dependent on understanding what an


organization is and isn't capable of. Prior to forming a strategic plan,
which of the following questions should an organization ask itself to help
determine its capabilities?

How many vendors do the organization's competitors use?

Will the employees enjoy and support the proposed changes?

Will employees receive a pay increase if the proposed changes are implemented?

Will the organization's leadership and management style support the proposed
changes?

https://docs.google.com/forms/d/e/1FAIpQLSe_EAdn9T3cnNgY1MBr4Yi0RS0utxLuIAYLclmRVChiUWq3TQ/viewscore?viewscore=AE0zAgCMmUqMl… 97/106 https://docs.google.com/forms/d/e/1FAIpQLSe_EAdn9T3cnNgY1MBr4Yi0RS0utxLuIAYLclmRVChiUWq3TQ/viewscore?viewscore=AE0zAgCMmUqMl… 98/106


‫ م‬10:40 2023/‫‏‬5/‫‏‬13 ASQ CMQ-OE : Practice Test #07 ‫ م‬10:40 2023/‫‏‬5/‫‏‬13 ASQ CMQ-OE : Practice Test #07

1/1* As part of the annual business planning process, the Quality Manager 1/1* When constructing a questionnaire for customer satisfaction, the main
submitted a revised Quality Improvement (QI) plan-with a budget-to be advantage to using a Likert-type format over a checklist format (Yes/No)
launched within the upcoming year. To improve the effectiveness of the is:
total quality system, one Ql project that was approved is the purchase
and implementation of a Quality Improvement System (QIS) software
The Likert format is easier for customers to use.
package to replace the current system, which contains numerous
database areas and uncontrolled spreadsheets, reports, and other Ql- Likert-type format scales are always odd (usually a 5- or 7-point scale) and the O
related documents. In addition, the current system is hard to search, center of the scale is always neutral, so the customer can express that they neither
making it difficult for new improvement teams or auditors to retrieve agree nor disagree with the statement in question.

specific information. The Likert format provides more response variability, giving customers a
chance to express a wider range of their agreement or disagreement with each
The Quality Manager agreed to be the project manager for the initiative. statement.
Her financial acumen and general management skills are essential for the
project to succeed. It was estimated that the project would take 30 weeks The data from a checklist can be analyzed using an ANOVA, whereas Likert data
cannot.
from planning to project close.

The project is beginning the design process with the selected QIS vendor.
To ensure user acceptance, which approach should be used in this stage
of the project? 1/1* Several assembly lines are moving from one facility to another. A defined
schedule has been created with a firm deadline. The best planning tool to
Knowledge, Experience, Skills, Aptitude, and Attitude (KESAA) method use would be a(n):

Plan-Do-Check-Act (PDCA) method


Tree diagram.
Gap analysis method
Prioritization matrix.
Requirements and testing method
Process Decision Program Chart (PDPC).

Activity Network Diagram (AND).

https://docs.google.com/forms/d/e/1FAIpQLSe_EAdn9T3cnNgY1MBr4Yi0RS0utxLuIAYLclmRVChiUWq3TQ/viewscore?viewscore=AE0zAgCMmUqMl… 99/106 https://docs.google.com/forms/d/e/1FAIpQLSe_EAdn9T3cnNgY1MBr4Yi0RS0utxLuIAYLclmRVChiUWq3TQ/viewscore?viewscore=AE0zAgCMmUqM… 100/106


‫ م‬10:40 2023/‫‏‬5/‫‏‬13 ASQ CMQ-OE : Practice Test #07 ‫ م‬10:40 2023/‫‏‬5/‫‏‬13 ASQ CMQ-OE : Practice Test #07

1/1* A team leader is coaching a team member to build further competence 0/1* Whose quality philosophy and principles overtly states that a company
by suggesting that the team member take on a new task like one she must eliminate what was commonly known as Management by
completed before. This task would primarily enhance the team member's: Objectives (MBO)?

Skill. Dr. Joseph Juran

Aptitude. Genichi Taguchi

Knowledge. Phillip B. Crosby

Experience. Dr. W. Edwards Deming

‫اإلجابة الصحيحة‬

Dr. W. Edwards Deming


0/1* A company wants to obtain ISO 9001 certification and senior
management needs to get every employee on board with completing
production paperwork by the end of the shift. One team leader cannot ‫التعليقات‬
consistently complete the paperwork due to additional responsibilities that Deming's 14 Points include statements about elimination of numerical quotas for the
she also must complete prior to the end of the shift. This employee workforce and numerical goals for management. Such quotas and goals tended to be
arbitrary and were often set as an attempt to motivate people to try harder rather than to
requests that a co-worker, who has time at the end of the shift, take on motivate conducting the necessary process improvements it takes to achieve them. It
some of her additional responsibilities, thereby giving her time to
.closely ties with his emphasis on blaming the process, not the people
complete the paperwork as needed.

This is an example of:

1/1* A multimillion-dollar project is showing signs of instability due to a lack of


A change agent.
resources and limited internal technical capability. Which of the following
Change management. is likely the reason why this is happening?

Breakthrough improvement.
The project was launched without considering the potential contribution to the
Coaching. financial statement at the micro-level

The project was approved using Return on Investment (ROI) analysis and the
‫اإلجابة الصحيحة‬
likelihood of critical financial gains
Change management. The interdepartmental project plan and budget were not properly allocated to
ensure that support needs were available when needed

The project's business case was too high-level with an emphasis on "what sells
‫التعليقات‬
to management
Removing obstacles to the ISO initiative is an essential aspect of managing change, which

.in this case is a more ISO- compliant environment

https://docs.google.com/forms/d/e/1FAIpQLSe_EAdn9T3cnNgY1MBr4Yi0RS0utxLuIAYLclmRVChiUWq3TQ/viewscore?viewscore=AE0zAgCMmUqM… 101/106 https://docs.google.com/forms/d/e/1FAIpQLSe_EAdn9T3cnNgY1MBr4Yi0RS0utxLuIAYLclmRVChiUWq3TQ/viewscore?viewscore=AE0zAgCMmUqM… 102/106


‫ م‬10:40 2023/‫‏‬5/‫‏‬13 ASQ CMQ-OE : Practice Test #07 ‫ م‬10:40 2023/‫‏‬5/‫‏‬13 ASQ CMQ-OE : Practice Test #07

0/1* During a quarterly financial review meeting, the Chief Financial Officer 0/1* As a key organizational leader with prior experience championing teams,
presented unflattering numbers for the Shipping Department. In you have been tasked with implementing a change management
response, the head of the Shipping Department accused the CFO of approach for the organization. The organization's core business is
trying to put all the blame on Shipping and a disagreement ensued. The providing call center services to companies in the U.S. and Canada. Last
Quality Manager calmly interjected and suggested that a more in-depth year, the organization began marketing and sales efforts to acquire
review of the numbers be done to identify and understand the related clients in Europe and China, and three new call centers have opened as
causes. In this situation, the Quality Manager demonstrated what skill? a result with more to follow.

Top management wants the change management approach to


Taking ownership accomplish the following:

Facilitation 1. Transition all call centers to an empowered, natural work team


structure.
Empowering
2. Identify and develop change agents within the call center workforce
Active listening that will drive continual improvement. 3. Establish buy-in on performance
metrics that will drive center efficiency and a high-degree of caller
‫اإلجابة الصحيحة‬ satisfaction. You are given the resources to hire an external consultant as
Facilitation a change agent for the initiative.

The change has been in place, and working well, for the past 6 months.
Local change agents are helping with continual improvement as planned.
‫التعليقات‬
The 6-month milestone was designated as the time to conduct an
The manager diffused the conflict using facilitation skills to suggest everyone look at data objective and honest evaluation of progress. Findings will determine any

.before drawing conclusions necessary system-level adjustments needed across all call centers.
Which statement reflects the most likely reason why subsequent systemic
changes could make things worse rather than better?

Due to systemic changes across all centers, local change agents lose power and
influence

Systemic changes could negatively impact local ownership

Decisions generated by feedback may never happen

Due to cultural differences and groupthink, staff in the new centers will not be
as open-minded to change as staff in the U.S. and Canada

‫اإلجابة الصحيحة‬

Systemic changes could negatively impact local ownership

‫التعليقات‬

https://docs.google.com/forms/d/e/1FAIpQLSe_EAdn9T3cnNgY1MBr4Yi0RS0utxLuIAYLclmRVChiUWq3TQ/viewscore?viewscore=AE0zAgCMmUqM… 103/106 https://docs.google.com/forms/d/e/1FAIpQLSe_EAdn9T3cnNgY1MBr4Yi0RS0utxLuIAYLclmRVChiUWq3TQ/viewscore?viewscore=AE0zAgCMmUqM… 104/106


‫ م‬10:40 2023/‫‏‬5/‫‏‬13 ASQ CMQ-OE : Practice Test #07 ‫ م‬10:40 2023/‫‏‬5/‫‏‬13 ASQ CMQ-OE : Practice Test #07
As time goes on, the change must become more standardized and efficient at a systemic
level. For example, the change should be supported by common Standard Operating
Procedures (SOP), tools, and IT resources, which potentially impact each team's sense of 0/1* The training director is updating the professional enhancement training
local control and may cause them to feel a loss of ownership that was put in place for the module for middle managers relating to the continuous improvement

.first 6 months
program. The previous program was deemed ineffective and needed
modification for practical application. What should the training director
propose?

Develop a new module with a hands-on workshop


1/1* You are an experienced Black Belt and have just been hired by a
manufacturing company which you now find out is in serious financial Focus on the selection of the trainer with the most expertise

trouble. Your boss is the Vice President of Manufacturing, who knows


Develop criteria for the selection of participants for each module
almost nothing about Six Sigma but has heard that Black Belts can
deliver millions of dollars in savings. He has told you that you have 2 Focus on strategic planning linkage with measurable initiatives
months to save $200,000. How would you evaluate his directive?
‫اإلجابة الصحيحة‬

You need to judge the level of support of the executive group and should convene O Focus on strategic planning linkage with measurable initiatives
a meeting to describe Six Sigma, what it can do, and the necessary leadership
commitment

You likely cannot do this alone and should initiate Green Belt training for others to ‫التعليقات‬
help you achieve the 2-month directive
This is the most logical choice because the module is about continuous improvement. Not
knowing the path to link with strategic planning is like a sailboat without a rudder; without
To meet the two-month objective, start working immediately on several of the the ability to steer along a certain course, the ship will sail aimlessly. Without
obvious areas needing improvement .measurement, it is difficult, if not impossible to detect change or improvement

While there may be much "low-hanging fruit" from which to choose, ask for
clarification about priorities so you can work on projects that best assist the
company

* When a team is first established, it goes through a period of growth referred


to as forming. This stage includes:

Questioning the progress of the team and the style of the team leader.

Reemphasizing or establishing additional team ground rules.

Excessive debate amongst members even when they mostly agree on the same
issues.

Discussions to clarify the purpose, goal, and necessary tasks for the project.

‫ سياسة الخصوصية‬- ‫ شروط الخدمة‬- .Google ‫لم يتم إنشاء هذا المحتوى وال اعتماده من ِقبل‬

 ‫نماذج‬

https://docs.google.com/forms/d/e/1FAIpQLSe_EAdn9T3cnNgY1MBr4Yi0RS0utxLuIAYLclmRVChiUWq3TQ/viewscore?viewscore=AE0zAgCMmUqM… 105/106 https://docs.google.com/forms/d/e/1FAIpQLSe_EAdn9T3cnNgY1MBr4Yi0RS0utxLuIAYLclmRVChiUWq3TQ/viewscore?viewscore=AE0zAgCMmUqM… 106/106

You might also like